Leadership test 1

Réussis tes devoirs et examens dès maintenant avec Quizwiz!

Primary nursing

A system in which one nurse is responsible for total care of a number of clients 24 hours a day, 7 days a week Provides comprehensive, individualized, and consistent care Associates provide care, but the primary nurse plans and coordinates care

Biopatch

Blue to the sky

Accuchecks

Every 6 hours for TPN

Authoritarian

Makes the decisions for the group Directive or autocratic Negative connotations

Delegation

Right circumstances Right task Right person Right direction/communication Right supervision

Distal Port

Used for viscous fluids

Chest X-Ray

After PICC/central line to verify placement before administering fluids

Fib/Flutter treatment

Anti platelet to prevent clots Beta Blocker and calcium channel blocker Digoxin

A charge nurse tells a new nurse, "You really need to get your skills up to speed." The statement hurts and embarrasses the new nurse. How can she best handle the situation? a) Tell the charge nurse she needs to be more specific about what she means. b) Tell the charge nurse she feels hurt by her statement. c) Discuss her feelings with a coworker in order to vent. d) Ask for a private meeting to explore the charge nurse's concerns in detail.

Ask for a private meeting to explore the charge nurse's concerns in detail. Correct Explanation: The charge nurse's statement is vague; the priority issue is to gather information about what she meant. Meeting privately with the charge nurse is one way to diffuse tension in a nonthreatening manner and gather information that might have professional value for the nurse. Stating that the nurse felt hurt immediately focuses on subjective issues rather than objective concerns. Professional respect dictates inquiring about what the charge nurse meant, rather than telling her to be more specific. Discussing the situation with a coworker may make the nurse feel better but doesn't address the issue at hand.

Nursing Delivery Models Total Patient Care

Case method Earliest model of nursing care Private-duty nurses Client centered Client has consistent contact with one nurse during shift

CLABSI Directly related to Patient Safety Goal #7

Central Line Associated Blood Stream Infections

Clamping

Check for air leaks or briefly to change units X-ray everyday, check before clamping

Challenges for management

Conflict resolution Organizational and Political Power Formal v. Informal Power Authority Motivation Time management

The primary responsibility of the nurse manager is to a) Provide direct client care b) Direct discharge planning c) Evaluate nursing care plans d) Create plans and budgets

Create plans and budgets Correct Explanation: Information about trends is necessary to project human and financial resource needs and to create plans and budgets. Nurse managers must be able to develop budgets that provide sufficient resources to accomplish unit goals

Which of the following has affected the way in which health care is delivered? a) Increased length of stay b) Increased hospital revenues c) DRGs d) Lack of alternative health care delivery systems

DRGs Correct Explanation: Hospitals and other health care providers are now reimbursed at a fixed rate for patients who have the same diagnosis as defined by diagnosis-related groups (DRGs). Decreasing hospital revenues, alternative health care delivery systems, and decreased length of stay have affected the ways in which health care is delivered

Patient Care

Delegate care when appropriate Cluster care Plan ahead

Interruptions

Each time you are stopped in the middle of one activity to give attention to something else, you are wasting valuable time How can you approach interruptions to save time?

Functional method

Evolved from concepts of scientific management Focuses on jobs to be completed Task-oriented approach Disadvantage is fragmentation of care

Bellows

Expand when suction is correct- adjust wall suction

Most Common sites

External jugular- most preferred site- less infection Subclavian- risk for puncturing lung Femoral- dirtiest- least preferred

Labs for TPN

Glucose

Goals

Goals provide a guide, a time frame, and a way to measure accomplishments

Concentrations

Greater than 10 % glucose or 4.2% amino acids put into central line Put filter at the end of TPN, closest to pt Connect lipids below the the filter

Lipids

Hang 10-12 hours

During the orientation to the hospital, the staff development educator discusses unit and institutional-based policies. What is the source of the practice rules that result in unit and institutional-based policies? a) Healthcare institution b) State legislation c) Board of nursing d) Federal legislation

Healthcare institution Correct Explanation: The healthcare institution determines the unit and institutional policies. These policies may vary from institution to institution. Such policies may include clinical procedures, policies specific to the institution, and personnel and employment policies.

Managing Goals

Identify objectives to be achieved Describe specific activities necessary to achieve these objectives Estimate time required for each activity Determine planned activities for concurrent action versus sequential Identify activities that can be delegated

Nursing Leadership

Leadership occurs when someone influences others to act Leaders are not necessarily representatives of the organization Managers are assigned their roles Leaders achieve their roles

Case management

More recent model of nursing care delivery Pioneered at the New England Medical Center Used in: Insurance-based programs, Employer-based health programs, Worker's compensation, Maternal-child health, Mental health, Hospital-based programs

Central Venous Catheters

Nontunneled central lines Tunneled central lines i.e. Hickman Broviac Peripherally Inserted Central Catheter (PICC)

After reporting to work for a night shift, Nurse L. learns that the unit will be understaffed because two RNs called out sick. As a result, each nurse on the unit will need to provide care for an additional four acute patients in addition to his or her regular patient assignment. Which of the following statements is true of Nurse L. when working in understaffed circumstances? a) Nurse L. is legally obliged to refuse to provide care when understaffing creates the potential for unsafe conditions. b) Nurse L. must document that float staff, nurses on overtime, and part-time staff were contacted in an effort to fill the gaps in care. c) Understaffing constitutes an extenuating circumstance that creates a temporarily lower expectation for care for Nurse L. d) Nurse L. will be legally held to the same standards of care as when staffing levels are normal.

Nurse L. will be legally held to the same standards of care as when staffing levels are normal. Correct Explanation: The claim of being overworked does not constitute a legal defense, and both the potential for liability and standards of care remain unchanged despite an increased patient assignment. While it is prudent to make all realistic attempts to fill the gaps in staffing, documenting these efforts does not change Nurse L.'s legal position. A nurse has the right to refuse an unsafe patient assignment but the nurse is not legally obliged to withhold care.

TPN/Lipids

Ordered a day before use Change all tubing with each new bag 10% glucose if run out Refrigerate and remove an hour prior to use

A nursing instructor is reviewing the parts of an EKG strip with a group of students. One student asks about the names of all the EKG cardiac complex parts. Which of the following items are considered a part of the cardiac complex on an EKG strip? Choose all that apply. a) P-R interval b) S-Q segment c) P wave d) T wave e) QRT wave

P wave • P-R interval • T wave Explanation: The EKG cardiac complex waves include the P wave, the QRS complex, the T wave, and possibly the U wave. The intervals and segments include the PR interval, the ST segment, and the QT interval

Paperwork

Plan and schedule times for paperwork Sort paperwork for effective processing Analyze paperwork frequently Do not be a paper shuffler; handle paper once

A nurse-manager works for a nonprofit health care corporation whose revenues have significantly exceeded annual expenses. The nurse-manager has been told to anticipate which action? a) Dividing revenue among stockholders as dividends b) Receiving a portion of the revenue to improve client services on the unit c) Reducing operating expenses to help the organization pay taxes on the revenue d) Identifying revenue as profit

Receiving a portion of the revenue to improve client services on the unit Correct Explanation: In a nonprofit organization, revenue exceeding expenses is tax-exempt and is usually reinvested in the organization and used to improve services. A for-profit organization calls

A busy nurse is working with a UAP (unlicensed assistive personnel). What tasks can the nurse appropriately delegate to the UAP? Mark all that apply. a) Assess the client?s risk for pressure ulcers. b) Administer routine oral medications. c) Record the client?s intake and output. d) Assist the client to the bedside commode. e) Assess the client?s need for education.

Record the client?s intake and output. • Assist the client to the bedside commode. Correct Explanation: It is crucial for the nurse to be aware of the legalities of delegation to unlicensed assistive personnel. Appropriate delegation to a UAP would include recording intake and output and assisting the client to the bedside commode. Assessment of the client's educational needs and the risk for pressure ulcers fall only under the nurse's scope of practice. Administering oral medications is not appropriate for unlicensed personnel.

A nurse manager must provide care to her clients and her employees under the constraints of her resources. This involves maintaining the financial constraints of her division. What term best describes this scenario? a) Management b) Financial management c) Resource management d) Allocation

Resource management Explanation: In modern healthcare organizations, managers usually are responsible for accomplishing the work of the unit within the constraints of available resources. This process is called resource management

Yellow label

TPN/Lipids High alert- 2 RNs check Keep covered to protect from light

A type of nursing which includes individualized care provided by the same nurse throughout the period of care is defined as a) case management. b) primary nursing. c) functional nursing. d) patient-focused nursing.

b

A nurse is teaching first aid to counselors of a summer camp for children with asthma. This is an example of what aim of health teaching? a Promoting health b Preventing illness c Restoring health d Facilitating coping

b. Teaching first aid is a function of the goal to prevent illness. Promoting health involves helping patients to value health and develop specific health practices that promote wellness. Restoring health occurs once a patient is ill, and teaching focuses on developing self-care practices that promote recovery. When facilitating coping, nurses help patients come to terms with whatever lifestyle modification is needed for their recovery or to enable them to cope with permanent health alterations.

The nurse and a student are discussing entry into the profession of nursing. Which of the following statements should the nurse use to describe a diploma program? a) "It is obtained by a 4-year program at a university." b) "It is obtained through a 2-year program at a university." c) "It is obtained by a 36-month program at a community college." d) "It is obtained by a 24-month program at a hospital."

"It is obtained by a 24-month program at a hospital." Explanation: Diploma programs are obtained through a hospital program and take 24 to 36 months. Associate degrees are obtained through a community college and take 2 academic or calendar years. Baccalaureate degrees are obtained through a 4-year degree at a senior college or university.

A nurse-educator is discussing healthcare reform trends during hospital orientation for a group of nursing fundamentals students. The nurse is correct in stating which of the following? a) "Distinctive to the United States is the dominance of the public element over the private one." b) "Spending on medical services will rise to almost 32% of the U.S. gross domestic product by 2021." c) "Systems are in place to pay for performance and penalize hospitals for excessive readmissions." d) "The United States is secnd in the world in total health care dollars spent annually."

"Systems are in place to pay for performance and penalize hospitals for excessive readmissions." Explanation: Health care trends already include paying for performance (HEDIS, HCAHPS) and penalizing hospitals for excess readmissions. In the United States, private insurers dominate over public, unlike in most countries. The United States is first in health care spending worldwide, and it is estimated that 20% of the gross domestic product will be spent on medical services by 2021

A nurse-educator is discussing healthcare reform trends during hospital orientation for a group of nursing fundamentals students. The nurse is correct in stating which of the following? a) "Distinctive to the United States is the dominance of the public element over the private one." b) "Systems are in place to pay for performance and penalize hospitals for excessive readmissions." c) "The United States is secnd in the world in total health care dollars spent annually." d) "Spending on medical services will rise to almost 32% of the U.S. gross domestic product by 2021."

"Systems are in place to pay for performance and penalize hospitals for excessive readmissions." Explanation: Health care trends already include paying for performance (HEDIS, HCAHPS) and penalizing hospitals for excess readmissions. In the United States, private insurers dominate over public, unlike in most countries. The United States is first in health care spending worldwide, and it is estimated that 20% of the gross domestic product will be spent on medical services by 2021.

Which statement by a staff nurse on the orthopedic floor indicates the need for further staff education? a) "Monitoring skin integrity is important while the continuous passive motion device is in place." b) "Bleeding is a complication associated with the continuous passive motion device." c) "The continuous passive motion device can decrease the development of adhesions." d) "The client is receiving physical therapy twice per day, so he doesn't need a continuous passive motion device."

"The client is receiving physical therapy twice per day, so he doesn't need a continuous passive motion device." Explanation: Further staff education is needed when the nurse states that the continuous passive motion device isn't needed because the client receives physical therapy twice per day. The continuous passive motion device should be used in conjunction with physical therapy because the device helps prevent adhesions. Bleeding is a complication associated with the continuous passive motion device; skin integrity should be monitored while the device is in use.

A student nurse observes a nurse case manager coordinating discharge for a patient diagnosed with congestive heart failure (CHF). Which of the following statements made by the patient indicates to the student that the patient understands the role of case manager? a) "The nurse case manager organized my daily nursing care during my hospitalization and arranged for the dietitian to teach me the importance of following a diet low in sodium." b) "The nurse case manager contacted my insurance company and has arranged for the home health nurse and physical therapist to visit me as soon as I get home." c) "The nurse case manager worked with my physician to coordinate my admission from his office to the hospital." d) "The nurse case manager arranged to have a wheelchair waiting to take me to my room. I was so short of breath I could not walk very far."

"The nurse case manager contacted my insurance company and has arranged for the home health nurse and physical therapist to visit me as soon as I get home." Correct Explanation: Coordination of care between nurses, other health care personnel, and insurance companies are roles of the nurse case manager. Nurse case managers coordinate patient care from the time of hospital admission to the time of discharge and often following discharge from an acute care setting. Care coordination provided by the nurse care manager is not episodic.

A nursing student is caring for one of the nurse's assigned cardiac clients. The student asks, "How can I tell the difference between sinus rhythm and sinus bradycardia when I look at the EKG strip" The best reply by the nurse is which of the following? a) "The P-R interval will be prolonged in sinus bradycardia, and you will have to measure carefully to note the width." b) "The P waves will be shaped differently." c) "The only difference is the rate, which will be below 60 bpm in sinus bradycardia." d) "The QRS complex will be smaller in sinus bradycardia."

"The only difference is the rate, which will be below 60 bpm in sinus bradycardia." Explanation: All characterestics of sinus bradycardia are the same as those of normal sinus rhythm, except for the rate, which will be below 60 in sinus bradycardia.

A nurse is conducting a class about health care trends for a group of newly graduated registered nurses at the community hospital. Which of the following six trends stated by the nurse reflects the Institute of Medicine's (IOM) focus? a) "The system should be safe, efficient, centralized, wellness promoting, government driven, and patient centered." b) "The system should be safe, effective, efficient, patient centered, timely, and equitable." c) "The system should be static, reliable, timely, equitable, efficient, and illness focused." d) "The system should be flexible, patient centered, bureaucratic, timely, safe, and efficient."

"The system should be safe, effective, efficient, patient centered, timely, and equitable." Correct Explanation: A safe, effective, efficient, patient-centered, timely, and equitable system is what is envisioned by the IOM. A bureaucratic, government-driven, illness-focused system is not advocated by the IOM

The nurse is caring for a 56-year-old male patient who had an implantable cardioverter defibrillator (ICD) implanted 2 days prior. The patient tells the nurse "My wife and I can never have sex again now that I have this ICD." The nurse's best response is which of the following? a) "Sex is permitted following the implantation of an ICD." b) "You really should speak to your wife about your concerns." c) "You seem apprehensive about resuming sexual activity." d) "I will be sure to share your concerns with the physician."

"You seem apprehensive about resuming sexual activity." Explanation: The patient treated with an electronic device experiences not only lifestyle and physical changes but also emotional changes. At different times during the healing process, the patient may feel angry, depressed, fearful, anxious, or a combination of these emotions. It is imperative for the nurse to observe the patient's response to the device and provide the patient and family members with emotional support and teaching as indicated. Identifying that the patient appears apprehensive about resuming sexual activity acknowledges the patient's concerns while allowing for further discussion. The remaining responses ignore the patient's feelings and do not facilitate an ongoing conversation or explore the patient's concern

The nurse is caring for a client who is being discharged after insertion of a permanent pacemaker. The client, an avid tennis player, is scheduled to play in a tournament in 1 week. What is the best advice the nurse can give related to this activity? a) "Cancel your tennis tournament and wait until fall, then try hockey; skating is much easier on pacemakers." b) "You will need to cancel this activity; you must restrict arm movement above your head for 2 weeks." c) "You may resume all normal activity in 1 week; if you are used to playing tennis, you may proceed with this activity." d) "You should avoid tennis; basketball or football would be a good substitute."

"You will need to cancel this activity; you must restrict arm movement above your head for 2 weeks." Explanation: It is important to restrict movement of the arm until the incision heals. The client should not raise the arm above the head for 2 weeks afterward to avoid dislodging the leads. The client must avoid contact sports (eg, basketball, football, hockey).

The nurse is caring for a client with atrial fibrillation. The client's symptoms started about 1 week ago, but he is just now seeking medical attention. The client asks the nurse why he has to wait several weeks before the cardioversion takes place. The best answer by the nurse is which of the following? a) "Your atrial chambers may contain blood clots now, so you must take an anticoagulant for a few weeks before the cardioversion." b) "There is a long list of clients in line to be cardioverted." c) "We have to allow your heart to rest for a few weeks before it is stressed by the cardioversion." d) "The doctor wants to see if your heart will switch back to its normal rhythm by itself."

"Your atrial chambers may contain blood clots now, so you must take an anticoagulant for a few weeks before the cardioversion." Explanation: Because of the high risk of embolization of atrial thrombi, cardioversion of atrial fibrillation that has lasted longer than 48 hours should be avoided unless the client has received warfarin for at least 3 to 4 weeks prior to cardioversion

Electrocardiogram (ECG) waveforms are printed on graph paper that is divided by light and dark vertical and horizontal lines at standard intervals. When the nurse is interpreting the heart rhythm, he or she understands that each large block equals how many seconds? a) 0.1 b) 0.4 c) 0.3 d) 0.2

0.2 Explanation: Each small block on the graph paper equals 0.04 second, and five small blocks form a large block, which equals 0.2 second.

3 levels of priority setting (guided by Maslow)

1. ABC, vital signs 2. mental status changes, untreated medical issues, acute pain, acute elimination problems, abnormal labs, risks 3. education, rest, coping, support

Dry Suction

1st chamber- drainage record drainage in output, mark date and time, and change unit when full Water seal chamber- left bottom- gentle tidaling in conjunction with respirations, bubbling indicates air leak suction control wet- upper left, set at 20, **controlled by the height of the water- control is the amount of suction

A 1-minute ECG tracing of a patient with a regular heart rate reveals 25 small square boxes within an RR interval. The nurse correctly identifies the patient heart rate as which of the following? a) 80 bpm b) 60 bpm. c) 70 bpm d) 100 bpm

60 bpm. Correct Explanation: A patient's HR can be obtained from the ECG tracing by several methods. A 1-minute strip contains 300 large boxes and 1500 small boxes. Therefore, an easy and accurate method of determining heart rate with a regular rhythm is to count the number of small boxes within an RR interval and divide by 1,500. In this instance, 1,500/25 = 60

A patient is admitted to the emergency department (ED) with complaints of chest pain and shortness of breath. The nurse notes an irregular rhythm on the bedside electrocardiograph (ECG) monitor. The nurse counts 9 RR intervals on the patient's 6-second rhythm tracing. The nurse correctly identifies the patient's heart rate as which of the following? a) 70 bpm b) 100 bpm c) 80 bpm d) 90 bpm

90 bpm Correct Explanation: An alternative but less accurate method for estimating heart rate, which is usually used when the rhythm is irregular, is to count the number of RR intervals in 6 seconds and multiply that number by 10. The RR intervals are counted, rather than QRS complexes, because a computed heart rate based on the latter might be inaccurately high. The same methods may be used for determining atrial rate, using the PP interval instead of the RR interval. In this instance, 9 × 10 = 90

A nurse leader is working with a client with hypertension who needs to learn about a low-sodium diet. The nurse recognizes the client is in the unfreezing stage of Lewin's Change theory with which client statement? a) ?I don?t see a problem with what I normally eat.? b) ?I am developing a daily meal plan for a low-sodium diet.? c) ?I understand why I need to eat a low-sodium diet.? d) ?I have selected my daily menu with low-sodium foods.?

?I understand why I need to eat a low-sodium diet.? Correct Explanation: Lewin?s Change theory involves three phases which include unfreezing, moving, and freezing. With the unfreezing stage, the person recognizes the need for change. The client who understands why a low-sodium diet is needed with hypertension recognizes that this is a necessary change for their health status. If there is not a problem recognized, as with continuing a normal diet, the person is not ready for change. The person who is planning a meal is in the moving phase, which is the initiation of change after planning. The person who has already made the change with their daily menu is in the refreezing stage, where change has been effectively made

A nurse hired to work in an ambulatory setting attends new employee orientation. The nurse never worked in ambulatory before and is concerned about the Scope and Standards of Practice for Professional Ambulatory Care Nursing. Which response, given by the nurse educator, would further explain the Scope and Standards of Practice for Professional Care Nursing to the new nurse? a) ?The Scope and Standards of Practice for Professional Ambulatory Care Nursing sets the standards for the nursing supervisor to assess a nurse.? b) ?The Scope and Standards of Practice for Professional Ambulatory Care Nursing takes precedent over the facility?s policies and procedures.? c) ?The Scope and Standards of Practice for Professional Ambulatory Care Nursing deal with the professional obligations of a nurse working in the ambulatory setting.? d) ?The Scope and Standards of Practice for Professional Ambulatory Care Nursing determines if a nurse is minimally competent to receive a license to practice as a nurse.?

?The Scope and Standards of Practice for Professional Ambulatory Care Nursing deal with the professional obligations of a nurse working in the ambulatory setting.? Explanation: The Scope and Standards of Practice for Professional Ambulatory Care Nursing are the standards of care for nurses working in the ambulatory arena. It does not take precedent over the facility?s policies and procedures, but must be worked in conjunction with the policies and procedures. It is not used for assessing nurses. NCLEX determines if a nurse is minimally competent to practice as a nurse

The nurse participates in a quality assurance program and reviewing evaluation data from the previous year. Which of the following does the nurse recognize as an example of outcome evaluation? a) A 97% bed occupancy rate in the critical care areas; 92% bed occupancy rate in the non-critical care areas. b) A 4% increase in the number of baccalaureate prepared nurses are employed in the facility. c) 98% of all hospital admissions had a nursing history completed within 24 hours of admission d) A 2% reduction in the number of repeat admissions for clients who underwent hip replacement surgery.

A 2% reduction in the number of repeat admissions for clients who underwent hip replacement surgery. Correct Explanation: Quality assurance programs focus on three types of evaluation: structure, process, and outcome. Outcome evaluation focuses on measurable changes in the health status of clients, such as a 2% reduction in the number of repeat admissions for clients who underwent hip replacement surgery. Structure evaluation focuses on the environment in which care is provided, such as the number of baccalaureate-prepared nurses employed in the facility and bed occupancy rates. Process evaluation focuses on the nature and sequence of activities carried out by nursing implementing the nursing process, such as 98% of all hospital admissions had a nursing history completed within 24 hours of admission.

Which patient would a nurse correctly refer to Medicare services? a) A 66-year-old patient with diabetes b) A low-income family with infants needing immunizations c) A patient with cancer d) A patient with a disability

A 66-year-old patient with diabetes Explanation: Medicare is a federal health care plan for individuals over the age of 65. An individual with a disability might be referred to Medicare, but other federal organizations must be involved for this to occur. A low-income family with infants needing immunizations would be referred to Medicaid. A client with cancer option is not the correct answer because the age of the individual is not noted to be over 65

Which client's care may a registered nurse (RN) safely delegate to the nursing assistant? a) A client with end-stage chronic obstructive pulmonary disease receiving patient controlled analgesia. b) A client requiring assistance ambulating, who was admitted with a history of seizures. c) A client with a trach that requires intermittent suctioning. d) A client who requires continuous pulse oximetry monitoring admitted with bronchitis.

A client requiring assistance ambulating, who was admitted with a history of seizures. Explanation: The RN may safely delegate assistance ambulated the client with a history of seizures to a nursing assistant. The RN should provide direct care to the client who requires continuous pulse oximetry monitoring because pulse oximetry interpretation requires assessment skills. Care of the clients requiring suctioning and patient-controlled analgesia can be safely delegated to a licensed practical nurse

The nurse is assigned the following client assignment on the clinical unit. For which client does the nurse anticipate cardioversion as a possible medical treatment? a) A client with third-degree heart block b) A client with atrial dysrhythmias c) A client with poor kidney perfusion d) A new myocardial infarction client

A client with atrial dysrhythmias Explanation: The nurse is correct to identify a client with atrial dysrhythmias as a candidate for cardioversion. The goal of cardioversion is to restore the normal pacemaker of the heart, as well as, normal conduction. A client with a myocardial infarction has tissue damage. The client with poor perfusion has circulation problems. The client with heart block has an impairment in the conduction system and may require a pacemaker.

Case management Defined as a

A collaborative process with several parts to meet an individual's health needs to promote quality, cost effective outcomes Integration of a multiple disciplines and services in coordinating care during client's span of illness Role includes collaboration, coordination, information processing, and information exchange Case managers serve as patient advocates

The licensed practical nurse is setting up the room for a client arriving at the emergency department with ventricular arrhythmias. The nurse is most correct to place which of the following in the room for treatment? a) A suction machine b) Cardioversion equipment c) A defibrillator d) An ECG machine

A defibrillator Explanation: The nurse is most correct to place a defibrillator close to the client room if not in the room. The nurse realizes that clients with ventricular dysrhythmias are at a high risk for fatal heart dysrhythmia and death. A suction machine is used to remove respiratory secretions. Cardioversion is used in a planned setting for atrial dysrhythmias. An ECG machine records tracings of the heart for diagnostic purposes. Most clients with history of cardiac disorders have an ECG completed

Which of the following nurse leaders most clearly exemplifies transformational leadership? a) A leader who is dynamic and inspiring and promotes change by the power of her convictions b) A leader who prioritizes the fact that every member of the team is considered valuable and equal c) A leader who prioritizes the delegation of leadership to the individual members of the group d) A leader who is careful to thoughtfully assess the issues and priorities surrounding a problem prior to acting

A leader who is dynamic and inspiring and promotes change by the power of her convictions Correct Explanation: Transformational leaders can create revolutionary change. They are often described as charismatic and they are unique in their ability to inspire and motivate others. A careful examination of contextual factors is associated with situational leadership while delegation to group members and an emphasis on equality are typical of laissez-faire and democratic leadership

The nurse is describing the role of the nurse entrepreneur. Which of the following statements best describe this role? a) A nurse responsible for adminstration of resources b) A nurse who manages a health-related business c) A nurse who teaches in a clinical setting d) A nurse who manages personnel involved in client care

A nurse who manages a health-related business Correct Explanation: The nurse entrepreneur usually has an advanced degree and manages a clinic or health-related business, conducts research, provides education, or serves as an adviser or consultant to institutions, political agencies, or businesses

The nurse is assigned to care for the following patients admitted to a telemetry unit. Which patient should the nurse assess first? a) A patient returned from an electrophysiology (EP) procedure 2 hours ago complaining of constipation b) A patient diagnosed with new onset of atrial fibrillation requiring scheduled IV Cardizem c) A patient whose implantable cardioverter defibrillator (ICD) fired twice on the prior shift requiring amiodarone IV d) A patient who received elective cardioversion 1 hour ago with a heart rate (HR) is 115 bpm

A patient whose implantable cardioverter defibrillator (ICD) fired twice on the prior shift requiring amiodarone IV Explanation: The patient's ICD that has fired on the previous shift should be seen first. This patient is in need of antidysrhythmic medication and this is the priority intervention. The remaining patients should be seen after this patient and are in no acute distress

Prioritizing - Urgent and Important

A patient's condition becomes life threatening and you have other patients who need your care How do you handle the situation?

A community health nurse provides a client with information about a local support group for those with multiple sclerosis. Providing this information is an example of which of the following? a) Conferring. b) A referral. c) A consultation. d) Reporting.

A referral. Correct Explanation: Referring is the process of sending or guiding a client to another source for assistance. Consultation is the process of inviting another professional to evaluate the client and make recommendations about his or her treatment. Conferring is to consult with someone to exchange ideas or seek information, advice, or instructions. Reporting is the oral, written, or computer-based communication of client data to others

After many years of advanced practice nursing, a nurse has recently enrolled in a nurse practitioner (NP) program. This nurse has been attracted to the program by the potential after graduation to provide primary care for clients, an opportunity that is most likely to exist in which of the following settings? a) A community hospital. b) A long-term care facility. c) A university hospital. d) A rural health center.

A rural health center. Correct Explanation: Many rural health centers employ few healthcare providers, and primary care is often provided by an NP. An NP may provide care in a long-term care facility or hospital, but in these settings, the NP is less likely to be the provider of primary care to clients.

What is the priority assessment for the nurse when developing a plan of care for a client from a poverty culture? a) Access to care b) Access to health insurance c) Access to affordable housing d) Access to financial assistance

Access to care Correct Explanation: Poverty has long been a barrier to adequate health care. If clients cannot access health care, it does not matter if they have affordable housing, health insurance, or the need for financial assistance. It is not possible to create a plan of care with client involvement without adequate support and access to care.

A group of nurses working in a long-term care facility fail to keep the narcotic medications in a secure location. The nurses also fail to count the medications before and after each shift, as indicated by the institution's policies and procedures. These failures may result in what type of disciplinary action? a) Action against the state regulating body b) Action against the nurses' licenses c) Action against the facility's state license d) Action against the pharmacist's license

Action against the nurses' licenses Explanation: In institutions, most controlled substances must be kept secure and monitored closely in accordance with institutional and state regulations. Failure to do so may lead to disciplinary action against the nurse's license.

Democratic

Actively guides the group toward achieving the group goals Every group member participates in decision making Concern and consideration for each group member

A nurse is caring for a client with a fresh postoperative wound following a femoral-popliteal revascularization procedure. The nurse fails to routinely assess the pedal pulses on the affected leg, and missed the warning sign that the blood vessel was becoming occluded. The nurse manager is made aware of the complication and the nurse's failure to assess the client properly. What action should be taken by the nurse manager? a) Review the nurse's malpractice insurance policy. b) Address the nurse's omissions as negligent behavior. c) Reprimand the nurse for being forgetful. d) Ask the nurse whether the client refused the assessments.

Address the nurse's omissions as negligent behavior. Correct Explanation: Negligence refers to careless acts on the part of an individual who is not exercising reasonable or prudent judgment. It also refers to the failure to do something that a reasonable person (another nurse) would do

Which of the following nursing interventions is most likely to be allowed within the parameters of a protocol or standing order? a) Changing a patient's intravenous (IV) fluid from normal saline to 5% dextrose b) Administering a glycerin suppository to a constipated patient who has not responded to oral stool softeners c) Administering a beta-adrenergic blocker to a new patient whose blood pressure is high on admission assessment d) Changing a patient's advance directive after his prognosis has significantly worsened

Administering a glycerin suppository to a constipated patient who has not responded to oral stool softeners Correct Explanation: Standing orders and protocols often surround the management of bowel elimination. Modification of a patient's IV fluid or administration of a new antihypertensive are patient-specific interventions that are physician initiated. The care team cannot independently change a patient's advance directive

AFib flutter predisposition

Age, cardiomapathy, diabetes, smoker, CAD, HF, CHF

A 26-year-old Air Force staff sergeant is returning for diagnostic follow-up to the cardiologist's office where you practice nursing. Her Holter monitor strip reveals a heart rate with normal conduction but with a rate consistently above 105 beats/minute. What other conditions can cause this response in a healthy heart? a) Elevated temperature b) Shock c) Strenuous exercise d) All options are correct

All options are correct Correct Explanation: It occurs in clients with healthy hearts as a physiologic response to strenuous exercise, anxiety and fear, pain, fever, hyperthyroidism, hemorrhage, shock, or hypoxemia. There are a variety of causes that can create an elevated heart rate in an otherwise healthy heart. Fever is one cause. There are a variety of causes that can create an elevated heart rate in an otherwise healthy heart. Shock is one cause. There are a variety of causes that can create an elevated heart rate in an otherwise healthy heart. Strenuous exercise is one cause

A nurse is working within the managed care delivery model. Which of the following is true regarding managed care? a) All plans have the same values underlying the delivery of care. b) Their values are not reflected in the decision making. c) There are no conflicts between cost-effectiveness and respectful care. d) All systems reflect the values of efficiency and effectiveness.

All systems reflect the values of efficiency and effectiveness. Correct Explanation: All systems in the managed care delivery model reflect the values of efficiency and effectiveness. Different plans may have different values underlying the delivery of care. However, they all reflect the business plan values of efficiency and effectiveness. Their values are reflected in the decision making and the policy development of the organization. Value conflicts between cost-effectiveness and respectful care may be seen.

The nurse is caring for clients on a telemetry unit. Which nursing consideration best represents concerns of altered rhythmic patterns of the heart? a) Altered patterns frequently turn into life-threatening arrhythmias. b) Altered patterns frequently produce neurological deficits. c) Altered patterns frequently affect the heart's ability to pump blood effectively. d) Altered patterns frequently cause a variety of home safety issues.

Altered patterns frequently affect the heart's ability to pump blood effectively. Correct Explanation: The best representation of a nursing concern related to a cardiac arrhythmia is the inability of the heart to fill the chambers and eject blow flow efficiently. Lack of an efficient method to circulate blood and bodily fluids produces a variety of complications such as tissue ischemia, pulmonary edema, hypotension, decreased urine output, and impaired level of consciousness. The other options can occur with dysrhythmias, but the cause stemming from the altered pattern is the best answer

CVC Insertion and Care Protocol

Always performed as sterile procedure, change q 24 hours Settings include: O.R., Radiology, Designated unit (i.e. PICC procedure rooms), Patient Room Large vein used with catheter tip placed into superior vena cava Ultrasound generally used to verify vein for access CVC placement verified via fluoroscopy

Which of the following clinical events constitute areas of potential liability for the nurses involved? Select all that apply. a) An anemic patient experiences a febrile reaction to a transfusion of packed red blood cells. b) A patient experiences a seizure after a missed dose of his scheduled anticonvulsant. c) A confused patient experiences a fall because her bedrails were left in a lowered position. d) A patient with no known cardiac history suffers an unwitnessed cardiac arrest and dies. e) An elderly patient develops skin breakdown on his coccyx because he was turned infrequently.

An elderly patient develops skin breakdown on his coccyx because he was turned infrequently. • A confused patient experiences a fall because her bedrails were left in a lowered position. • A patient experiences a seizure after a missed dose of his scheduled anticonvulsant. Correct Explanation: Liability exists when the elements of duty, breach of duty, causation, and damages exist. Failure to turn an immobile patient, missing a dose of medication, and leaving a vulnerable patient's bedrails lowered all constitute breaches of nurses' duties that result in damages. Not every untoward event is evidence of liability; however, febrile blood reactions or unexpected cardiac arrests may occur without any inappropriate causation by care providers

Nationally notifiable infectious conditions

Anthrax, botulism, cholera, diptheria, hepatitis, HIV, influenza- associated pediatric mortality, meningoccal disease, shigellosis, and syphilis

Which of the following medication classifications is more likely to be expected when the nurse is caring for a client with atrial fibrillation? a) Diuretic b) Potassium supplement c) Anticoagulant d) Antihypertensive

Anticoagulant Explanation: Clients with persistent atrial fibrillation are prescribed anticoagulation therapy to reduce the risk of emboli formation associated with ineffective circulation. The other options may be prescribed but not expected in most situations

Conflict has emerged on a nursing unit due to the perception by new graduates that some of the more experienced nurses are manipulating the patient assignment to ensure a lighter workload during night shifts. How should the manager of the unit best address this conflict? a) Gather evidence over the next several weeks in order to determine if the practice is indeed happening b) Reassure the new graduates that the more experienced nurses are acting in the interests of both staff and patients c) Arrange a meeting where the issue can be discussed and addressed by as many of the nurses as possible d) Arrange for the newer nurses to organize the patient assignment for a trial period

Arrange a meeting where the issue can be discussed and addressed by as many of the nurses as possible Correct Explanation: Open, explicit, and participatory conflict resolution that is based on collaboration is an effective strategy for the management of conflict. Gathering evidence does not directly address the conflict that presently exists and reassurance may be unwarranted and false. Allowing the new graduates to create the patient assignment may perpetuate selfish practices and does not resolve animosity between the two camps.

The nurse is caring for a client who does not speak the same language. The UAP (unlicensed assistive personnel) speaks the same language as the client. What parts of communicating with the client could the nurse appropriately delegate to the UAP? Mark all that apply. a) Ask the client questions regarding personal care needs. b) Orient the client and family to the room, including the call light button. c) Provide teaching to the client, including discharge instructions. d) Demonstrate and teach new caregiving procedures to the family. e) Interview the client as part of the admission assessment. f) Counsel the client about making adjustments to a new medical condition.

Ask the client questions regarding personal care needs. • Orient the client and family to the room, including the call light button. Correct Explanation: Delegation to unlicensed assistive personnel requires knowledge of the RN role and what tasks can be legally delegated to the UAP. The RN can delegate asking clients questions about personal care needs and orientation to the room (for example, the call light button). It is inappropriate to have the UAP interview the client as part of the admission assessment, or provide teaching to the client, or provide teaching to the family, or counsel the client. Those duties are legally the role of the RN and would be most appropriately addressed with a professional translator interpreting for the nurse and the client

A nurse-manager has decided to delegate responsibility for the review and revision of the surgical unit's client-education materials. Which statement illustrates the best method of delegation? a) Ask the assistant manager to develop a plan for the review and revision of client-education materials. b) Tell the nursing staff they're responsible for the review and revision and welcome their recommendations for improving the materials. c) Ask the two most proficient staff nurses to form a task force to review and revise client-education materials within the next 6 weeks. Have these nurses solicit input from clients and staff members. d) Tell the nursing staff that the client education materials need revision. Ask the staff to select people to review the materials and make suggestions for change.

Ask the two most proficient staff nurses to form a task force to review and revise client-education materials within the next 6 weeks. Have these nurses solicit input from clients and staff members. Correct Explanation: Delegation must be clear and precise. The nurse-manager must assign responsibility, identify the task to be accomplished, explain the necessary outcomes, and define the time frame available to complete the work. The remaining options don't clearly define the work to be done, don't clearly assign responsibility or specify desired outcomes, or establish a time frame for completion of the task.

A nurse precepting second year nursing students from a community college plans clinical experiences for students who will most likely graduate in which timeframe? a) In 2 more years b) At the end of the year c) In 1 more year d) In 3 more years

At the end of the year Correct Explanation: Nursing students from a community college are most likely students attempting to obtain an associate degree, which is a 2-year program. Therefore, the nurse should plan clinical experiences for students who will most likely graduate at the end of the year, as this is their second year

The nurse caring for a patient with a dysrhythmia understands that the P wave on an electrocardiogram (ECG) represents what phase of the cardiac cycle? a) Early ventricular repolarization b) Ventricular depolarization c) Ventricular repolarization d) Atrial depolarization

Atrial depolarization Explanation: The P wave represents atrial depolarization. The QRS complex represents ventricular depolarization. The T wave represents ventricular repolarization. The ST segment represents early ventricular repolarization, and lasts from the end of the QRS complex to the beginning of the T wave

An 83-year-old resident in the long-term care facility where you practice nursing has an irregular heart rate of around 100 beats/minute. He also has a significant pulse deficit. What component of his history would produce such symptoms? a) Atrial flutter b) Bundle branch block c) Heart block d) Atrial fibrillation

Atrial fibrillation Correct Explanation: In atrial fibrillation, several areas in the right atrium initiate impulses resulting in disorganized, rapid activity. The atria quiver rather than contract, producing a pulse deficit due to irregular impulse conduction to the AV node. The ventricles respond to the atrial stimulus randomly, causing an irregular ventricular heart rate, which may be too infrequent to maintain adequate cardiac output. Atrial fibrillation generally causes disorganized activity, irregular heart rates, and pulse deficits. It is not atrial flutter. Atrial fibrillation generally causes disorganized activity, irregular heart rates, and pulse deficits. It is not heart block. Atrial fibrillation generally causes disorganized activity, irregular heart rates, and pulse deficits. It is not bundle branch block

A patient's ECG tracing reveals an atrial rate between 250 and 400, with saw-toothed P waves. The nurse correctly identifies this dysrhythmia as which of the following? a) Atrial fibrillation b) Ventricular fibrillation c) Atrial flutter d) Ventricular tachycardia

Atrial flutter Correct Explanation: The nurse correctly identifies the ECG tracing as atrial flutter. Atrial flutter occurs in the atrium and creates impulses at a regular atrial rate between 250 and 400 times per minute. The P waves are saw-toothed in appearance. Atrial fibrillation causes a rapid, disorganized, and uncoordinated twitching of atrial musculature. The atrial rate is 300 to 600, and the ventricular rate is usually 120 to 200 in untreated atrial fibrillation. There are no discernible P waves. Ventricular fibrillation is a rapid, disorganized ventricular rhythm that causes ineffective quivering of the ventricles. The ventricular rate is greater than 300 per minute and extremely irregular, without a specific pattern. The QRS shape and duration is irregular, undulating waves without recognizable QRS complexes. Ventricular tachycardia is defined as three or more PVCs in a row, occurring at a rate exceeding 100 beats per minute

A client presents to the emergency department via ambulance with a heart rate of 210 beats/minute and a sawtooth waveform pattern per cardiac monitor. The nurse is most correct to alert the medical team of the presence of a client with which disorder? a) Premature ventricular contraction b) Atrial flutter c) Asystole d) Ventricular fibrillation

Atrial flutter Explanation: Atrial flutter is a disorder in which a single atrial impulse outside the SA node causes the atria to contract at an exceedingly rapid rate. The atrioventricular (AV) node conducts only some impulses to the ventricle, resulting in a ventricular rate slower than the atrial rate, thus forming a sawtooth pattern on the heart monitor. Asystole is the absence of cardiac function and can indicate death. Premature ventricular contraction indicates an early electric impulse and does not necessarily produce an exceedingly rapid heart rate. Ventricular fibrillation is the inefficient quivering of the ventricles and indicative of a dying heart

Which medication is the drug of choice for sinus bradycardia? a) Cardizem b) Pronestyl c) Atropine d) Lidocaine

Atropine Correct Explanation: Atropine is the medication of choice in treating symptomatic sinus bradycardia. Lidocaine treats ventricular dysrhythmias. Pronestyl treats and prevents atrial and ventricular dysrhythmias. Cardizem is a calcium channel blocker and treats atrial dysrhythmias

A nurse is completing a shift assessment on a patient admitted to the telemetry unit with a diagnosis of syncope. The patient's heart rate is 55 bpm with a blood pressure of 90/66 mm Hg. The patient is also experiencing dizziness and shortness of breath. Which of the following medications will the nurse anticipate administering to the patient based on these clinical findings? a) Cardizem b) Lidocaine c) Pronestyl d) Atropine

Atropine Correct Explanation: The patient is demonstrating signs and symptoms of symptomatic sinus bradycardia. Atropine is the medication of choice in treating symptomatic sinus bradycardia. Lidocaine treats ventricular dysrhythmias. Pronestyl treats and prevents atrial and ventricular dysrhythmias. Cardizem is a calcium channel blocker and treats atrial dysrhythmias

On a medical unit, the nurses complain that they have no voice in the decisions that are made in the operation of the unit. The nurses state they are always told by the nurse manager to perform tasks instead of being asked. Which of these best describes the leadership style of the nurse manager? a) Laissez-faire b) Democratic c) Nondirectional d) Autocratic

Autocratic Correct Explanation: This nurse manager, by not soliciting staff feedback and telling the staff what to do rather than ask, would be demonstrating autocratic leadership. Democratic leadership involves equal sharing of power between the leader and the staff; everyone is working together toward a mutual vision and goals. Laissez-faire leadership, which is also known as nondirectional leadership, gives power to the group rather than the leader

A client is received into the emergency department after getting shot in the chest. The client is hemorrhaging profusely and is in hypovolemic shock. The nurse calls a code blue. What type of leadership style will be most effective during the management of the code? a) Democratic leadership b) Laissez-faire leadership c) Autocratic leadership d) Transactional leadership

Autocratic leadership Correct Explanation: Autocratic leadership will be most effective in this scenario. Autocratic leadership involves the leader assuming control over the decision and activities of the group. During code blue, a leader is needed to direct the actions needed and make quick decisions to positively affect the client. Laissez-faire leadership would ultimately lead to the demise of the client. Democratic leadership could possibly delay the client receiving life-sustaining measures and would be less effective than autocratic leadership. Transactional leadership would involve a reward system which would be unethical in this situation.

A nurse graduate applies for a job working in a hospital that achieved magnet status. Which conditions would this nurse expect? (Select all that apply.) a) Focus on positive patient care outcomes b) Higher levels of staff burnout and exodus from the bedside c) Autonomous, accountable professional nursing practice d) Higher staff turnover e) Supportive nurse managers f) Centralized decision making

Autonomous, accountable professional nursing practice • Focus on positive patient care outcomes • Supportive nurse managers Correct Explanation: A magnet status facility would focus on positive client care outcomes, value autonomous and accountable professional nursing practice, and be supportive of nurse managers. A magnet status facility would not have centralized decision making, rather focus on self-governance. A magnet status facility would have lower staff turnover and lower levels of staff burnout and low numbers of nursing staff leaving the bedside

A patient tells the nurse "my heart is skipping beats again; I'm having palpitations." After completing a physical assessment, the nurse concludes the patient is experiencing occasional premature atrial complexes (PACs). The nurse should instruct the patient to complete which of the following? a) Apply supplemental oxygen. b) Request sublingual nitroglycerin. c) Avoid caffeinated beverages. d) Lie down and elevate the feet.

Avoid caffeinated beverages. Explanation: If PACs are infrequent, no medical interventions are necessary. Causes of PACs include caffeine, alcohol, nicotine, stretched atrial myocardium (e.g., as in hypervolemia), anxiety, hypokalemia (low potassium level), hypermetabolic states (e.g., with pregnancy), or atrial ischemia, injury, or infarction. The nurse should instruct the patient to avoid caffeinated beverages

A 73-year-old client has returned to the postanesthesia care unit where you practice nursing. The client had a pacemaker implanted and it is your responsibility to begin client education upon his becoming alert. Which of the following postimplantation instructions must you provide to the client now that he has a permanent pacemaker? a) Delay for at least 3 weeks activities such as swimming and bowling. b) Keep the arm on the side of the pacemaker higher than the head. c) Avoid sources of electrical interference. d) Keep moving the arm on the side where the pacemaker is inserted.

Avoid sources of electrical interference. Explanation: The nurse must instruct the client with a permanent pacemaker to avoid sources of electrical interference, such as MRI devices, large industrial motors, peripheral nerve stimulators, etc. The main warning to a client with a pacemaker is to avoid sources of electrical interference

A nurse working with patients in the community is aware that which of the following is a true statement related to environmental factors in a community? a) The quality of air and water are relatively consistent when comparing urban and rural environments. b) Barriers to accessing healthcare within a community may include lack of transportation. c) Environmental factors focus on the harmful effects on an individual's health. d) Lack of health insurance is a negative environmental factor affecting one's access to healthcare.

Barriers to accessing healthcare within a community may include lack of transportation. Correct Explanation: Environmental barriers to accessing healthcare within a community include lack of transportation, distance to services, and location of services. Lack of healthcare insurance is an economic barrier to service, not an environmental barrier. The quality of air and water differ across communities and are not the same. Environmental factors not only focus on the harmful effects on health, but also include the helpful effects on health.

Which of the following tasks is most appropriate for the nurse to delegate to the unlicensed assistive personnel? a) Preparation of insulin for the diabetic client with elevated blood glucose b) Insertion of a urinary catheter in a client with benign prostatic hypertrophy c) Bed bath for the newly-admitted client who has with multiple skin lesions d) Ambulation of the client with a history of falls for the first time after surgery

Bed bath for the newly-admitted client who has with multiple skin lesions Explanation: Preparing insulin is outside of the UAPs scope of practice. The UAP may have the skills to insert an indwelling catheter and ambulating clients, but the clients involved each have qualifiers that complicate the tasks. The safest delegation is to have the UAP bathe the client with skin lesions and report any abnormal findings to the nurse

A nurse exits the room of a confused patient without raising the side rails on the bed. The failure to raise the side rails would be which of the following elements of liability related to malpractice? a) Damages b) Breach of duty c) Duty d) Causation

Breach of duty Correct Explanation: Breach of duty is failing to meet the standard of care, and in this case, it was the failure to execute and document the use of appropriate safety measures. Causation is the failure to use appropriate safety measures that results in injury to the patient. Duty refers to an obligation to use due care and is defined by the standard of care appropriate for the nursepatient relationship. Damages are the actual harm or injury resulting to the patient.

A 78-year-old client was just admitted to the cardiac step-down unit where you practice nursing. Upon stabilizing his condition, you begin a conversation about his symptoms and you answer his questions to the best of your ability. In your discussion of cardiac dysrhythmias, which of the following would you rule out as a likely origination point for cardiac dysrhythmias? a) Atria b) Atrioventricular node c) Bundle of His d) Ventricles

Bundle of His Explanation: Cardiac dysrhythmias may originate in the atria, atrioventricular node, or ventricles. They do not originate in the Bundle of His. Cardiac dysrhythmias do not originate in the Bundle of His

The nurse manager is holding a meeting with the nursing team to discuss management's decision to reduce staffing on the nursing unit. During the discussion, one of the staff nurses stands up and yells at the nurse manager, using profanity, and threatening "to take this decision further." To defuse this situation, which of the following would be the best step for the nurse manager to take? a) Ask the rest of the staff if they also feel the same way. b) Suspend the nurse who is acting out for the inappropriate behavior. c) Tell the nurse who is acting out to settle down and to act professionally. d) Call a break in the meeting and talk to the nurse in a private place.

Call a break in the meeting and talk to the nurse in a private place. Correct Explanation: When an individual is verbally acting out and others are present, it is advisable to isolate the individual by either removing him/her from the audience or removing the audience. Doing this gives the person an opportunity to regain control of rational thinking without embarrassment in front of his/her peers. It is not appropriate to initiate a suspension in public. By taking the person aside, it also keeps the audience from encouraging or coaching the acting-out individual and further escalating the situation. Asking the nurse to settle down and act professionally is not enough in this escalating circumstance.

Nursing students are reviewing information about expanded nursing roles. They demonstrate understanding of the information when they identify which of the following as roles of an advanced practice nurse? Select all that apply. a) Certified nurse-midwife b) Certified medical-surgical nurse c) Certified critical care nurse d) Nurse practitioner e) Clinical nurse specialist f) Certified registered nurse anesthetist

Certified nurse-midwife • Nurse practitioner • Clinical nurse specialist • Certified registered nurse anesthetist Explanation: Advanced practice nurses are nurses with advanced specialized education, usually at the graduate level. They include certified nurse midwives, nurse practitioners, clinical nurse specialists, and certified registered nurse anesthetists. Certified critical care nurses and certified medical-surgical nurses have passed the certification examination but are not routinely considered advanced practice nurses.

Functions Planning

Choosing the organization's mission and vision Devising departmental goals Selecting strategies to achieve goals Deciding on the allocation of resources Example: Budgeting

A nurse fails to administer a medication that prevents seizures, and the patient has a seizure. The nurse is in violation of the Nurse Practice Act. What type of law is the nurse in violation of? a) Federal b) Civil c) Supreme d) Criminal

Civil Correct Explanation: Malpractice cases are generally the kind of civil cases that involve nurses.

A nurse is reviewing the medical record of a client to determine his eligibility for home care services. Which of the following would alert the nurse to a potential problem in this area? a) Client is discharged from the facility after open heart surgery. b) Client requires sterile dressing changes to an abdominal wound. c) Client requires instructions about admininstering insulin injections. d) Client attends social groups at a senior center three times a week.

Client attends social groups at a senior center three times a week. Explanation: To be eligible for home care services, a client must be acutely ill, homebound, and in need of skilled nursing services. Attending social activities at a senior center three times a week suggests that the client may not be homebound. Sterile wound care and teaching about insulin administration are considered skilled nursing activities. Discharge after open heart surgery indicates that the client is experiencing an acute illness or condition

A 73-year-old client is due to return from surgery where he has had an automatic implanted cardioverter defibrillator (AICD) implanted. Understanding the purpose of the AICD, you indicate to his family why the implantation is positive for the client. Which of the following would be the least likely reason for the client's AICD implantation? a) Client experiences recurrent episodes of atrial flutter. b) Client is at risk for death due to structural cardiac disease with poor ventricular function. c) Client has survived at least one episode of cardiac arrest with ventricular origin. d) Client experiences recurrent episodes of ventricular tachycardia.

Client experiences recurrent episodes of atrial flutter. Correct Explanation: AICDs are used for dysrhythmias of a ventricular nature. AICDs are used for dysrhythmias of a ventricular nature. It is not about cardiac arrest. AICDs are used for dysrhythmias of a ventricular nature. It is not about ventricular tachycardia. AICDs are used for dysrhythmias of a ventricular nature. It is not about structural cardiac disease

Case managers rely on many tools to plan and coordinate client care. Which of the following are commonly used tools? a) Standards of care, HIPAA regulations, and electronic medical records b) Practice guidelines, outcome data, and staff rosters c) Clinical pathways, practice guidelines, and standards of care d) Clinical pathways and drug formularies

Clinical pathways, practice guidelines, and standards of care Correct Explanation: Case managers make use of tools such as clinical pathways, practice guidelines, and standards of care to help them plan and coordinate care. Hospitals and insurance companies may develop their own or rely on published protocols for guidance. Drug formularies are not tools used by case managers in planning and coordinating care. HIPAA regulations and EMRs are not tools used by case managers in planning and coordinating care. Outcome data and staff rosters are not tools used by case managers in planning and coordinating care.

Jennifer is a nurse manager who is trying to resolve a conflict between the day and night shifts. She wants to convince the involved persons to set aside their differences, determine a priority common goal having to do with improved patient care, and accept mutual responsibility for achieving this goal. Jennifer is using which of the following types of conflict resolution? a) Avoiding b) Competing c) Compromising d) Collaborating

Collaborating Correct Explanation: Collaborating has all parties set aside previously determined goals, determine a priority common goal, and accept mutual responsibility for achieving this goal. This requires mutual respect, honest communication, and shared decision making. Avoidance, as a conflict resolution, involves looking the other way and doing nothing to resolve the conflict. Competing has one side winning at the expense of the other. Compromising means that each side gives up something of value.

Derrick is the nurse manager for the psychiatric unit. There are major conflicts between the day and night shift staff. Derrick suggests that each shift put aside their differences for a time and determine a common major goal. Which of the following conflict resolution styles does Derrick display? a) Avoiding b) Competing c) Smoothing d) Collaborating

Collaborating Correct Explanation: With collaborating there is a joint effort to resolve the conflict with a win-win solution. All parties set aside previously determined goals, determine a priority common goal, and accept mutual responsibility for achieving this goal. With avoiding there is awareness of the conflict situation but the parties involved decide to either ignore the conflict or avoid/postpone its resolution. Competing is an approach that results in a win for one party at the expense of the other. Smoothing is an effort to complement the other party and focus on agreement rather than disagreement thus reducing the emotion in the conflict. The original conflict is rarely resolved with this technique

Carol is considering applying for a leadership role on her medical unit. She asks her colleagues what they consider to be good personal leadership skills. Which of the following would be appropriate? Choose all that apply. a) Knowledge of all nursing b) Religious values c) Communication d) Self-evaluation e) Problem solving

Communication • Self-evaluation • Problem solving Correct Explanation: Communication, problem solving, and self-evaluation are three of the four basic nursing leadership skills, but are not all inclusive. Religious values are personal and may contribute to a person's strength, but are not included in what is needed for a public leader. No one nurse can ever know everything there is to know about nursing.

What role will the nurse have when admitting a client to a hospital for outpatient surgery that will result in discharge the same day? a) Schedule the client for screening tests. b) Provide detailed information on the procedure. c) Prepare for long-term care needs. d) Complete regular admission procedures.

Complete regular admission procedures. Correct Explanation: Clients entering the hospital setting for outpatient surgery have regular admission procedures conducted by the nurse. Scheduling of screening tests and initial teaching is completed in the days prior to the surgery. Same-day surgery and discharge may require community-based follow-up, but it generally does not require long-term care. Detailed information on the procedure will be provided by the physician performing the procedure

A nurse manager and the charge nurse have two employees who are disagreeing with the accomplishment of client care. The manager determines a mutually agreed-upon time to discuss this situation with all the members involved. This activity is a) Delegation b) Conflict resolution c) Problem solving d) Client protection

Conflict resolution Correct Explanation: To produce high-quality care, the manager must constantly be developing the staff to meet new challenges. Along with their managers, nurses engage in conflict resolution.

A nurse manager notices that a number of medication errors have occurred on the unit with nurses giving hydralazine instead of hydroxyzine. What would be the most appropriate action for the nurse manager? a) Post the names of nurses making medication errors on a bulletin board b) Provide an inservice on the differences between hydralazine and hydroxyzine c) Require nurses to retrieve individual doses of the medications from the pharmacy d) Consult with pharmacy to ensure distinct labeling of the medications

Consult with pharmacy to ensure distinct labeling of the medications Explanation: Sometimes medication errors increase with drugs that are similar in name. These are sometimes referred to as SALAD names, which refer to sound-alike, look-alike drugs. The pharmacy should be consulted to help determine a way to label each medication that draws attention to the name of the medication.

A nurse assesses a client with psychotic symptoms and determines that the client needs vest restraints. However, the client asks the nurse not to put on vest restraints. What would be the best nursing action? a) Apply wrist restraints instead of vest restraints. b) Restrain the client with vest restraints. c) Apply restraints after giving a sedative. d) Contact the physician and obtain necessary orders.

Contact the physician and obtain necessary orders. Correct Explanation: If a nurse feels that a client needs to be restrained, the nurse should inform the physician and obtain necessary orders. The nurse should also discuss this with the client's family members and ask their opinion. The nurse should not sedate the client and then restrain him, as the nurse could be charged with battery if he or she restrains the client without orders. Applying a wrist restraint instead of a vest restraint is like compromising with the client, which is unethical.

Nurses in various health care settings provide services to prevent the fragmentation of care that is occurring as a health care trend in today's society. What role of the nurse is most important in preventing this effect? a) Care provision b) Teacher c) Counselor d) Coordinator of care

Coordinator of care Explanation: The most important role of the nurse in preventing fragmentation of care would be coordinator of care. Care coordination is the deliberate organization of patient care activities between two or more participants (including the patient) involved in a patient's care to facilitate the appropriate delivery of healthcare services. The roles of care provider, counselor, and teacher are all important roles, but the priority role is as the coordinator of care

Positive pressure

Creates tension Pneumothorax

Prioritization

Deciding which needs or problems require immediate attention and which ones can be delayed and for how long. Process that envisions client outcomes Includes predicting problems if another choice is made instead Weigh possible future events, the time to accomplish, and the associated tasks Know the PURPOSE of the care, 5 whys Know the current PICTURE and desired PICTURE Know PART for each team member Know to PLAN priorities

A nurse manager of a hospital unit is working within a decentralized management structure. Which nursing action best exemplifies this type of system? a) Nurses are not intimately involved in decisions involving patient care. b) Senior managers make all the decisions. c) Decisions are made by those who are most knowledgeable about the issue. d) Nurse managers are not accountable for patient, staffing, supplies, or budgets.

Decisions are made by those who are most knowledgeable about the issue. Correct Explanation: The best example of a nurse manager of a hospital unit working within a decentralized management structure would be that decisions are made by those who are most knowledgeable about the issue. Nurses would be greatly involved in decisions involving client care. Senior managers would not make all the decisions within a decentralized management structure. Nurse manages could be accountable for client, staffing, supplies, and/or budgets

The nurse manager who asks staff members to give suggestions on how to improve collaboration between nurses and physicians is exhibiting what style of leadership? a) Laissez-faire b) Transactional c) Autocratic d) Democratic

Democratic Correct Explanation: The nurse manager who involves staff members in decision-making by asking for suggestions is exhibiting a democratic style of leadership. Autocratic leadership style would be exhibited by a leader who gave staff little opportunity to give feedback or suggestions. Transactional leadership also involves little input by staff into decision-making and is instead focused on providing rewards when tasks are completed. A leader with a laissez-faire style would give all control to staff members to determine how to improve collaboration.

In anticipation of discharge, a nurse is teaching the daughter of an elderly woman how to change the dressing on her mother's venous ulcer. Which of the following teaching strategies is most likely to be effective? a) Demonstrate and explain the procedure, and then have the daughter perform it. b) Explain the procedure clearly and slowly while providing multiple opportunities for the daughter to ask questions. c) Use a multimedia strategy that combines animation with narration. d) Provide explicit written and verbal instructions, and ask the daughter to explain back to the nurse how she would perform the dressing change.

Demonstrate and explain the procedure, and then have the daughter perform it. Correct Explanation: All steps of a procedure such as a dressing change should be demonstrated, practiced, and provided in writing. The client or caregiver should then perform the procedure or treatment in the presence of the nurse to demonstrate his or her understanding and ability to carry out the procedure. This is more likely to facilitate success than providing a passive multimedia resource, explaining, or providing written instructions alone without reciprocal demonstration

Which of the following is the best example of the nurse implementing the ANA standard relating to diagnosis? a) Determining a client is at risk for falls b) Evaluating a client's laboratory results c) Assessing a client's apical pulse rate d) Teaching a diabetic client to inject insulin

Determining a client is at risk for falls Correct Explanation: When the nurse is implementing the ANA standards relating to diagnosis, the nurse analyzes the assessment data, such as laboratory results and apical pulse rate to determine a nursing diagnosis or issue to be addressed by nursing. Determining a client is at risk for falls is an example of the nurse identifying/diagnosing a nursing issue

The nurse manager on an orthopedic unit has determined that the nurses are not keeping the nursing diagnoses up to date on patient care plans and, in turn, the nurses are not using the plan of care. What is a feasible approach to correcting this problem? a) Provide an in-service on interviewing and physical assessment skills and discussing the importance of these skills with the staff. b) Request that a staff development nurse instruct the nurses on concept mapping for use instead of care planning. c) Develop a process for periodic review of care plans that focuses on deleting and updating the nursing diagnoses. d) Delegate the updating of nursing diagnoses for all patient's on the unit to one nurse for each shift.

Develop a process for periodic review of care plans that focuses on deleting and updating the nursing diagnoses. Explanation: Upon recognizing that the nursing diagnoses are not up to date, an effective approach by the nurse manager is to establish a process for periodic review of the plan of care. This review process will require deletion of nursing diagnoses that have been resolved and adding new diagnoses as needed. Implementing concept mapping will not correct the problem of poorly updated nursing diagnoses, as concept mapping requires the identification of nursing diagnoses. Developing interviewing and assessment skills is an important component of the assessment phase of the nursing process. Also, one nurse should not be responsible for updating nursing diagnoses for all patient care plans on the unit.

Home health care is one of the most rapidly growing areas of the health care system. What are chief tasks of the home health care nurse? (Select all that apply.) a) Providing for a dignified death at home b) Administering medications c) Providing patient teaching and counseling d) Providing continuity of care e) Collecting payment for nursing care f) Developing a nursing care plan

Developing a nursing care plan • Providing patient teaching and counseling • Providing continuity of care • Administering medications Explanation: The chief tasks of the home health care nurse are developing a nursing care plan, providing client teaching and counseling, providing continuity of care, and administering medications. The home health care nurse is not tasked with collecting payment for nursing care. The home health care nurse is not tasked with providing for a dignified death at home. This is the role of the hospice nurse

A nurse-manager appropriately behaves as an autocrat in which situation? a) Planning vacation time for staff b) Identifying the strengths and weaknesses of a client-education video c) Evaluating a new medication-administration process d) Directing staff activities if a client experiences a cardiac arrest

Directing staff activities if a client experiences a cardiac arrest Correct Explanation: In a crisis situation, the nurse-manager should take command for the benefit of the client. Planning vacation time and evaluating procedures and client resources require staff input and are actions characteristic of a democratic or participative manager.

ER

Dislodged tube, place in sterile water and call for help to get new set up If dislodged, place petroleum gauze with tape on 3 sides

Elective cardioversion is similar to defibrillation except that the electrical stimulation waits to discharge until an R wave appears. What does this prevent? a) Disrupting the heart during the critical period of ventricular repolarization b) Disrupting the heart during the critical period of atrial repolarization c) Disrupting the heart during the critical period of ventricular depolarization d) Disrupting the heart during the critical period of atrial depolarization

Disrupting the heart during the critical period of ventricular repolarization Correct Explanation: It is similar to defibrillation. One difference is that the machine that delivers the electrical stimulation waits to discharge until it senses the appearance of an R wave. By doing so, the machine prevents disrupting the heart during the critical period of ventricular repolarization. Therefore, options A, C, and D are incorrect.

A patient states that his recent fall was caused by the fact that his scheduled antihypertensives were mistakenly administered by two different nurses, an event that is disputed by both of the nurses identified by the patient. Which of the following measures should the nurses prioritize when anticipating that legal action may follow? a) Consult with practice advisors from the state board of nursing. b) Liaise with the hospital's legal department as soon as possible. c) Document the patient's claims and the events surrounding the alleged incident. d) Enlist support from nursing and nonnursing colleagues from the unit.

Document the patient's claims and the events surrounding the alleged incident. Correct Explanation: It is imperative for nurses to carefully and accurately document assessment findings and the nursing care that they provide, a fact that is especially salient when legal action is anticipated. This thorough and accurate assessment should precede consultation with the legal department, the state board of nursing, and colleagues

The nursing student asks the home health nurse what data is required for a Medicare home plan of care. Which item would be incorrect for the nurse to include when responding to the student? a) Types of services and equipment required b) Documented need for a speech pathologist c) Functional limitations d) Medications and treatments

Documented need for a speech pathologist Correct Explanation: A documented need for a speech pathologist is not required for a Medicare home plan of care. Required data would include types of services and equipment required, functional limitations of the client, and ordered medications and treatments

The nursing student asks the home health nurse what data is required for a Medicare home plan of care. Which item would be incorrect for the nurse to include when responding to the student? a) Types of services and equipment required b) Documented need for a speech pathologist c) Medications and treatments d) Functional limitations

Documented need for a speech pathologist Correct Explanation: A documented need for a speech pathologist is not required for a Medicare home plan of care. Required data would include types of services and equipment required, functional limitations of the client, and ordered medications and treatments

The nurse is preparing a patient for upcoming electrophysiology (EP) studies and possible ablation for treatment of atrial tachycardia. Which of the following information should the nurse include? a) The procedure will occur in the operating room under general anesthesia. b) The procedure takes less time than a cardiac catheterization. c) After the procedure, the arrhythmia will not recur. d) During the procedure, the arrhythmia will be reproduced under controlled conditions.

During the procedure, the arrhythmia will be reproduced under controlled conditions. Correct Explanation: During EP studies, the patient is awake and may experience symptoms related to the arrhythmia. EP studies do not always include ablation of the arrhythmia.

The nurse and student nurse are observing a cardioversion procedure completed by a physician. At which time is the nurse most correct to identify to the student when the electrical current will be initiated? a) During ventricular depolarization b) During stimulation of the SA node c) During the QRS complex d) During repolarization of the heart

During ventricular depolarization Explanation: The electrical current is initiated at the R wave when ventricular depolarization occurs. The electrical current completely depolarizes the entire myocardium with the goal of restoring the normal pacemaker of the heart. The other options focus on an incorrect timing that will not restore the normal electrical conduction

A patient with a history of mitral stenosis is admitted to the intensive care unit (ICU) with the abrupt onset of atrial fibrillation. The patient's heart rate ranges from 120 to 140 bpm. The nurse recognizes that interventions are implemented to prevent the development of which of the following? a) Embolic stroke b) Myocardial infarction c) Heart failure d) Renal failure

Embolic stroke Explanation: Intervention is implemented to prevent the development of an embolic event/stroke. Patients with a history of previous stroke, transient ischemic attack (TIA), embolic event, mitral stenosis, or prosthetic heart valve and who develop atrial fibrillation are at significant risk of developing an embolic stroke. Antithrombotic therapy is indicated for all patients with atrial fibrillation, especially those at risk of an embolic event, such as a stroke, and is the only therapy that decreases cardiovascular mortality. These patients are often placed on warfarin, in contrast to patients who have no risk factors, who are often prescribed 81 to 325 mg of aspirin daily

Managed care

Emphasizes communication and coordination of care among healthcare team members Attempts to integrate efficiency of care, access, and cost of care Critical pathways or multidisciplinary actions plan (MAP) Grids that outline critical events expected to happen during client's hospitalization Focus on high-cost, high-volume areas

A nurse who provides care on a postsurgical unit is performing discharge teaching as a component of her effort to ensure continuity of care. Which of the following is the primary goal of continuity of care? a) Controlling costs and maximizing patient outcomes after discharge from hospital b) Increasing patients' knowledge base and health maintenance behaviors c) Ensuring a smooth and safe transition between different healthcare settings d) Minimizing nurses' legal liability during patient transitions between healthcare institutions

Ensuring a smooth and safe transition between different healthcare settings Correct Explanation: Continuity of care exists primarily to ensure smooth, safe transitions so that patients may maximize recovery and health. Nurses build patients' knowledge bases in order to achieve this goal. Minimizing nurse liability and healthcare costs are not key justifications for continuity of care

A nurse who provides care on a post-surgical unit is performing discharge teaching as a component of her effort to ensure continuity of care. Which of the following is the primary goal of continuity of care? a) Ensuring a smooth and safe transition between different healthcare settings. b) Increasing clients' knowledge base and health maintenance behaviors. c) Controlling costs and maximizing client outcomes after discharge from the hospital. d) Minimizing nurses' legal liability during client transitions between healthcare institutions.

Ensuring a smooth and safe transition between different healthcare settings. Explanation: Continuity of care exists primarily to ensure smooth, safe transitions so that clients may maximize recovery and health. Nurses build clients' knowledge bases to achieve this goal. Minimizing nurse liability and healthcare costs are not key justifications for continuity of care

The nurse overhears two nursing assistants saying that they think it is ridiculous that a female Hispanic patient insists that only female nurses care for her. The nurse interprets this discussion as an example of which of the following? a) Subculture b) Cultural humility c) Ethnocentrism d) Cultural nursing assessment

Ethnocentrism Correct Explanation: Ethnocentrism occurs when one makes a value judgment on another culture from one's own cultural perspective. A subculture is a large group of people who share characteristics that identify them as a distinctive entity. Cultural humility includes the acknowledgement of one's own cultural knowledge deficits related to provision of culturally appropriate care. Cultural nursing assessment includes a systematic examination of a patient's cultural beliefs, values, and practices

Unused ports

Flush with 5ml/10 units Heparin every 24 hours Aspirate, flush w/ saline, med, saline, heparin Heparin prevents clots

You are caring for a client who has premature ventricular contractions. What sign or symptom is observed in this client? a) Hypotension b) Fever c) Fluttering d) Nausea

Fluttering Correct Explanation: Premature ventricular contractions usually cause a flip-flop sensation in the chest, sometimes described as "fluttering." Associated signs and symptoms include pallor, nervousness, sweating, and faintness. Symptoms of premature ventricular contractions are not nausea, hypotension, and fever

Bureaucratic leadership

Focus is on rules and regulations Hierarchy is prominent in decision making Data collection

Functions Organizing

Formal system of working relationships Identifying tasks and assigning them to individuals or teams Coordinating activities to meet unit goals Example: Staffing and scheduling Leading Power is an essential component of leading

A team leader chooses to apply the participative leadership style in managing the team of nurses. What value does this reflect? a) Freedom. b) Altruism. c) Justice. d) Equality.

Freedom. Explanation: The participative leadership style reflects the team leader's value for freedom. The team leader supports the right of other team members to suggest alternatives to the plan of care. This situation does not reflect the values of justice, altruism, or equality. When the team leader reports incompetent nursing practice objectively and factually, these actions reflect the value for justice. When the team leader assists others in providing care when they are unable to do so, it reflects altruism or a concern for the welfare of others. When the team leader interacts with the staff nurses in a non-discriminatory manner, the team leader promotes the value of equality

The nurse is working on a telemetry unit, caring for a client who has been in a sinus rhythm for the past 2 days with a heart rate of 88 to 96 beats per minute. The client puts on the call light in the bathroom and reports severe dizziness. The telemetry shows a heart rate of 46 beats per minute. What should the nurse be prepared to do? a) Assist with a temporary pacemaker. b) Send the client to the cardiac catheterization laboratory. c) Give an IV bolus of atropine. d) Prepare the client for maze surgery.

Give an IV bolus of atropine. Correct Explanation: Atropine 0.5 mg given rapidly as an intravenous bolus every 3 to 5 minutes to a maximum total dose of 3.0 mg is the medication of choice in treating symptomatic sinus bradycardia.

Minimize Routine Work

Group similar items within the divisions of the work shift Group routine tasks during the least productive time Use transition or waiting time productively Implement the daily plan and follow up

A home care nurse is scheduled to visit a client who lives in an area that is considered a high-crime area. Which of the following would be most appropriate? a) Have another home care nurse come along on the visit. b) Refuse to visit the client unless there is another member present. c) Ensure that the client has a working phone in the home. d) Schedule the visit as the first visit of the day.

Have another home care nurse come along on the visit. Explanation: Personal safety is essential. When making visits to areas that are considered unsafe such as high-crime areas, the home care nurse should have another home care nurse accompany him or her on the visit rather making the visit alone. Although making the visit early in the day, such as the first visit, may be helpful, having two people instead of one promotes greater safety. Ensuring a working phone is appropriate but would not help ensure the nurse's safety when in the client's neighborhood or when walking from the car to the client's home. Having another family member in the home during the visit also may be helpful, but it would not address the potential unsafe situation of the neighborhood

A charge nurse assesses a group of staff nurses as competent individually but ineffective and nonproductive as a team. How should the charge nurse address the staff nurses about her concerns? a) Have the staff nurses express their feelings and emotions. b) Incorporate the staff nurses in decision making. c) Increase staffing to prevent fatigue from overwork and understaffing. d) Ask the staff nurses if they feel unhappiness about the current leadership.

Have the staff nurses express their feelings and emotions. Correct Explanation: The most common reason for lack of productivity in a group of competent nurses is inadequate communication or unexpressed feelings and emotions. Unhappiness about leadership, fatigue from overwork and understaffing, and failure to incorporate staff in decision making could contribute to the problematic situation, but they're less likely to be the cause of the problem.

Which domain of perioperative nursing practice focuses on clinical processes and outcomes? a) Safety b) Health care systems c) Behavioral responses d) Physiological responses

Health care systems Correct Explanation: The health care system consists of structural data elements and focuses on clinical processes and outcomes. Safety, behavioral responses, and physiological responses reflect phenomena of concern to perioperative nurses and are composed of nursing diagnoses, interventions, and outcomes

A nurse is working as part of a team developing a community-based care program for the local homeless population. Which of the following would be important to keep in mind when developing the program? Select all that apply. a) Health problems are often related to a person's living situation. b) Most homelessness is chronic. c) Ill homeless people often deteriorate more rapidly than those who are not homeless. d) Homeless people often have no or insufficient health insurance. e) The homeless population is a relatively homogenous group.

Health problems are often related to a person's living situation. • Ill homeless people often deteriorate more rapidly than those who are not homeless. • Homeless people often have no or insufficient health insurance. Correct Explanation: The homeless population is a heterogeneous group and includes members of both dysfunctional and intact families, the unemployed, and those who cannot find affordable housing. Some people are temporarily homeless as a result of catastrophic natural disasters. Homeless people are often underinsured or uninsured and have limited or no access to health care. Many health problems are related in large part to living situation. Because of numerous barriers, homeless people seek health care late in the course of a disease and deteriorate more quickly than clients who are not homeless.

A 43-year-old male came into the emergency department where you practice nursing and was diagnosed with atrial fibrillation. It's now 48 hours since his admittance and the dysrhythmia persists. Which of the following medications will the client's healthcare provider most likely order? a) Flecainide (Tambocor) b) Warfarin (Coumadin) c) Dabigatran (Pradaxa) d) Heparin

Heparin Explanation: Heparin is generally prescribed initially if the dysrhythmia persists longer than 48 hours. Reference:

The nurse is planning discharge of the client who had surgery for a left hip replacement. The client is being discharged from the hospital to the home and requires home medical services. Which item would be provided by home medical services? a) Homemaking b) Pain management c) Intravenous therapy d) High-rise toilet seat

High-rise toilet seat Correct Explanation: Home medical services provide durable medical equipment, such as walkers, canes, crutches, wheelchairs, high-rise toilet seats, commodes, beds, and oxygen. Custodial services include homemaking and housekeeping services, as well as companionship and live-in services. Hospice services provide pain management, physician services, spiritual support, respite care, and bereavement counseling. High-technology pharmacology services provide intravenous therapy, home uterine monitoring, ventilator management, and chemotherapy.

A nursing facility has recently implemented new policies regarding nurse-to-patient ratios. The nursing staff seems very resistant to the change. How can the nurse manager help the staff accept the change? a) Challenge staff?s beliefs and values regarding providing quality patient care. b) Institute the changes immediately and collectively to decrease anticipation anxiety. c) Use the laissez-faire leadership style to address staff concerns. d) Hold a unit meeting to discuss how the changes will benefit staff.

Hold a unit meeting to discuss how the changes will benefit staff. Correct Explanation: Holding meetings allows open communication and opportunities for feedback. Listing the benefits of change to the individual and group also helps with buy-in. Laissez-faire leadership styles tend to increase conflicts and anxiety during times of change. Challenging staff beliefs will increase anxiety and create unpleasant work environments. Change should be instituted gradually, if possible

A client with pulmonary fibrosis is prescribed home oxygen therapy. Which health team member is responsible for evaluating the client's knowledge of home oxygen use? a) Hospital staff nurse b) Social worker c) Home health nurse d) Respiratory rehabilitation assistant

Home health nurse Correct Explanation: The home health nurse is responsible for evaluating the client's knowledge of home oxygen use. The social worker is responsible only for coordinating the services. The hospital staff nurse and physician do not observe the client in the home, so they can't adequately evaluate the client's knowledge of home oxygen use.

While assessing a client, the nurse finds a heart rate of 120 beats per minute. The nurse recalls that causes of sinus tachycardia include which of the following? a) Hypothyroidism and athletic training b) Digoxin and vagal stimulation c) Hypovolemia and fever d) Vagal stimulation and sleep

Hypovolemia and fever Correct Explanation: Causes of sinus tachycardia include physiologic or psychological stress (acute blood loss, anemia, shock, hypovolemia, fever, and exercise). Vagal stimulation, sleep, hypothyroidism, athletic training, and Digoxin all will cause a slow heart rate

Learning Outcomes

Identify time wasters. Identify goals. Set priorities. Group activities and minimize routine work. Manage personal organization and self-discipline. Minimize time wasters.

The selection of a nursing care delivery system (NCDS) is critical to the success of client care in a nursing area. Which factor is essential to the evaluation of an NCDS? a) Determining how planned absences, such as vacation time, will be scheduled so that all staff are treated fairly b) Identifying salary ranges for various types of staff c) Identifying who will be responsible for making client care decisions d) Deciding what type of dress code will be implemented

Identifying who will be responsible for making client care decisions Explanation: Determining who has responsibility for making decisions regarding client care is an essential element of all client care delivery systems. Dress code, salary, and scheduling planned staff absences are important to any organizations, but they are not actually determined by the NCDS

Cardioversion

If low BP, High RR, cool, clammy, high hR Adensosin- stops heart momentarily and heart re-establishes conduction- same as cardioversion have pt cough or bear down

Sara (RN) is the designated charge nurse on the telemetry unit. She organizes and facilitates the unit meetings; however, during most of the meetings, Betsy (RN) runs the show and influences staff decisions just by her charisma and personality. Betsy's power to lead is defined by which of the following terms? a) Situational b) Democratic c) Implied d) Explicit

Implied Correct Explanation: Implied power arises when a person who has no official assigned role, assumes a leadership role by virtue of the force of her/his personality and charisma. Sara's official assigned role gives her explicit power by virtue of that assignment. Situational and democratic are types of leadership rather than power

Laissez-faire

Inactive, passive, and permissive Offers few commands, questions, suggestions, or little criticism Relies on adult behaviors that are consistently accountable and responsible

The nurse working with the hospital's infection control team is attempting to decrease the transmission of health care-associated pathogens. Which of the following will be most effective? a) Revising the facility?s infection control protocols b) Incentivizing health care workers to utilize hand hygiene c) Encouraging visitors to adhere to isolation precautions d) Limiting visitors to family members over the age of 18

Incentivizing health care workers to utilize hand hygiene Explanation: Most health care-associated pathogens are transmitted via the contaminated hands of health care workers. Therefore, the most effective strategies for decreasing transmission are those that educate or encourage health care workers to utilize effective hand hygiene.

Team nursing

Individualized nursing care given to clients by a nursing team led by a professional nurse Members include RNs, LPNs, nursing assistants Responsible for care for an 8- or 12-hour shift Emphasizes humanistic values and individualized client care at a personal level Nurse leader motivates employees

Complications associated with CVC's

Infections Catheter occlusions due to either malposition or thrombosis Drug precipitates caused by incompatible drugs mixing to form precipitates within catheter Embolism Extravasation

A 63-year-old client is in the cardiac step-down unit where you practice nursing. In your discussions about his condition, the client is puzzled as to what causes the heart to be an effective pump. Which of the following statements would you include in your response? a) Inherent rhythmicity of cardiac muscle tissue b) Inherent electrons in muscle tissue c) Sufficient blood pressure d) Inherent rhythmicity of all muscle tissue

Inherent rhythmicity of cardiac muscle tissue Explanation: Cardiac rhythm refers to the pattern (or pace) of the heartbeat. The conduction system of the heart and the inherent rhythmicity of cardiac muscle produce a rhythm pattern, which greatly influences the heart's ability to pump blood effectively

A nurse manager notes an increase in the frequency of client falls during the last month. To promote a positive working environment, how would the nurse manager most effectively deal with this problem? a) Determine if client falls have increased on other nursing units in the hospital b) Reprimand the nursing personnel responsible for the clients when the falls occurred c) Investigate the circumstances that contributed to client falls d) Institute a new policy on the prevention of client falls on the unit

Investigate the circumstances that contributed to client falls Correct Explanation: The most effective method to address the increased frequency of client falls and to promote a positive working environment would be to determine the circumstances that contributed to the clients' falls. Attempting to identify and reprimand individual nurses does not lead to an atmosphere of openness and honesty in determining the causes. Instituting a new policy to prevent falls is premature before identifying why the falls are occurring. It may be relevant later to determine if other nursing units are having the same problem, but it is not necessary at this time.

s a nurse-manager of a medical-surgical unit reviews the month's risk-management data, she notices that a number of incident reports were completed because 6 p.m.(1800) medications were administered late. Dinner is served between 5:30 p.m. (1730) and 6 p.m. (1800). Staff take their dinner breaks between 5 p.m. (1700) and 6:30 p.m.(1830). Based on this information, which is the most appropriate action for the nurse-manager to take? a) Investigate when medications are given, staff and client dinner times, the number of medications that must be given at 6 p.m. (1800), and staff availability between 5 p.m. (1700) and 6 p.m. (1800). b) Terminate the nurses responsible for failing to administer medications on time. c) Decide that the staff must postpone dinner breaks until at least 7 p.m. (1900). d) Decide that the kitchen staff must change the time they deliver supper trays.

Investigate when medications are given, staff and client dinner times, the number of medications that must be given at 6 p.m. (1800), and staff availability between 5 p.m. (1700) and 6 p.m. (1800). Correct Explanation: An effective nurse-manager knows that to accurately evaluate risk-management findings, she must look at the entire process and the circumstances surrounding each incident. Terminating staff without such evaluation doesn't resolve all the problem's contributing factors. She shouldn't change dinner breaks or kitchen delivery times unless she has evaluated how these factors influence medication administration

Functions Directing

Involves human resource management such as Managing conflict, delegating, communication, and Disciplinary matters Example: Annual employee evaluations

Ask:

Is it life threatening or potentially life threatening if task is not done? Would another client be endangered if I did this task now? Is this task essential to client or staff safety? Is this task essential to the medical treatment or nursing POC?

The nurse discussing the importance of professional nursing organizations setting standards should include which of the following statements? a) It sets the standard for nursing research. b) It helps to identify nurses. c) It helps identify nursing as a profession. d) It helps regulate nursing licensure.

It helps identify nursing as a profession. Explanation: Professional nursing organizations that set standards for nursing helps nursing meet the defining criteria for a profession. However, they do not regulate nursing practice, set standards for nursing research, or help to identify nurses

A nursing student is giving to a client with heart failure a medication with a positive inotropic effect on the heart. The student asks what a "positive inotropic" effect is. The correct response would be which of the following? a) It increases the force of the myocardial contraction. b) It increases the respiratory rate. c) It increases the heart rate. d) It causes the kidneys to retain fluid and increase intravascular volume

It increases the force of the myocardial contraction. Correct Explanation: A positive inotropic effect increases the force of myocardial contraction. A positive chronotropic effect increases the heart rate. A positive inotropic effect will usually help slow respiratory rate and will increase blood flow through the kidneys, so fluid output will increase

The staff educator is teaching a class in dysrhythmias. What statement is correct for defibrillation? a) The client is sedated before the procedure. b) It is a scheduled procedure 1 to 10 days in advance. c) It is used to eliminate ventricular dysrhythmias. d) It uses less electrical energy than cardioversion.

It is used to eliminate ventricular dysrhythmias. Correct Explanation: The only treatment for a life-threatening ventricular dysrhythmia is immediate defibrillation, which has the same effect as cardioversion, except that defibrillation is used when there is no functional ventricular contraction. It is an emergency procedure performed during resuscitation. The client is not sedated but is unresponsive. Defibrillation uses more electrical energy (200 to 360 joules) than cardioversion.

The preoperative nurse is reviewing the chart of a client whose surgery is scheduled to begin in the next 15 minutes and notices that the consent form is not signed. The nurse contacts the surgeon who states, "We have already reviewed this procedure extensively, so ask the client to sign the consent form and I will verify it in the operating room." Which action by the nurse is most appropriate? a) Ask the client to sign the consent; witness the signature and inform the operating room staff of the modification in the procedure. b) Send the client to the operating room and inform the staff that the consent form needs to be signed. c) Keep the client in the preoperative area and inform the surgeon that it is the physician's responsibility to obtain consent for surgery. d) Ask the operating room staff to delay the procedure until the consent is signed.

Keep the client in the preoperative area and inform the surgeon that it is the physician's responsibility to obtain consent for surgery. Correct Explanation: The responsibility for securing informed consent from the client lies with the person who will perform the procedure. This is usually the physician. The nurse may sign as a witness, signifying that the client signed the consent form without coercion and was alert and aware of the act

A nurse is working with a student nurse who is caring for a client with an acute bleeding cerebral aneurysm. Which action by the student nurse requires further intervention? a) Administering I.V. fluid as ordered and monitoring the client for signs of fluid volume excess b) Keeping the client in one position to decrease bleeding c) Maintaining the client in a quiet environment d) Positioning the client to prevent airway obstruction

Keeping the client in one position to decrease bleeding Correct Explanation: The student nurse shouldn't keep the client in one position. She should carefully reposition the client often (at least every hour). The client needs to be positioned so that a patent airway can be maintained. Fluid administration must be closely monitored to prevent complications such as increased intracranial pressure. The client must be maintained in a quiet environment to decrease the risk of rebleeding

The nurse manager calls a staff into a unit meeting to discuss patient satisfaction. During the meeting, several staff members assume control. The nurse manager does not intervene to regain control of the group. Which type of leadership style is the nurse embodying? a) Autocratic b) Quantum c) Democratic d) Laissez-faire

Laissez-faire Correct Explanation: Laissez-faire leadership style involves the leader relinquishing power or control to the group. Democratic leadership style involves sharing the decision making process and activities with others who have an interest. Autocratic leadership style involves assuming control over the decisions and activities of the group. Quantum leadership style involves seeing an organization and members as interconnected and collaborative. This style involves change as continually unfolding, and frequently incorporates technology.

A nurse is scheduled to perform an initial home visit to a new client who is beginning home intravenous therapy. As the nurse is getting out of her car and beginning to approach the client's building, a group of men begin following and jeering at her. Which of the following is the nurse's best response to this situation? a) Leave the area in her car, provided she can get to it safely. b) Perform the home visit and ensure that the group is gone before she leaves. c) Call out to attract attention from bystanders. d) Confront the group of men in an assertive but non-aggressive manner.

Leave the area in her car, provided she can get to it safely. Correct Explanation: The nurse's safety is paramount, and the nurse's best response to a perceived threat when performing a home visit is to remove herself from the situation, provided this can be achieved without incurring further risk

Mandatory Reporting

Legal requirement to report act, event, or situation that is designated as reportable by state or local law Certain types of health information Certain vital statistics Communicable diseases Actual or suspected abuse Abuse or neglect of minors, older adults Good faith immunity Nurses in violation of NPA Certain injuries or illnesses Other examples of mandatory reporting (rape, assault, sexual assault, kidnapping, possessing firearm on school property, indecent liberties with minor)

Indications for insertion:

Long term antibiotics Chemotherapy Frequent labs (blood tests) Total parenteral nutrition (TPN)/ Lipids

The nurse is asked to develop an in-service to explain documents guiding professional nursing practice on the obstetrical unit. One of the documents included is the Code of Ethics. The nurse correctly explains that the Code of Ethics asks nurses to demonstrate which behaviors? Select all that apply. a) Maintain integrity and shape social policy. b) Develop, maintain, and improve health care environments. c) Be responsible and accountable for individual practice. d) Increase professional competence and personal growth. e) Ask the hospital for fair compensation for work.

Maintain integrity and shape social policy. • Increase professional competence and personal growth. • Be responsible and accountable for individual practice. • Develop, maintain, and improve health care environments. Correct Explanation: The Code of Ethics describes those actions by the nurse that guide their practice. It is the responsibility of each nurse to be active in determining policy for health care for all citizens and assuring that the way nursing is practiced is of the highest caliber. Nursing needs to participate in the development of health care of the future, while caring for all members of society. In order to be productive in shaping policy, nurses need to be politically astute while growing personally and professionally to meet the needs of clients. The Code of Ethics does not address compensation for work

Care includes

Maintaining sterility Dressing changes as indicated by status and/or policy Maintaining patency Verifying placement via measurements Ensuring stability of placement Central lines are sutured in place, PICC lines are not and need to be measured Swab caps are replaced every day or with dressing change, always keep clamped

When a person works to prevent relapse and to sustain the gains made from the actions taken, he or she is in which stage of the Transtheoretical Model of Change? a) Contemplative b) Termination c) Action d) Maintenance

Maintenance Explanation: A person is in the maintenance stage of the Transtheoretical Model of Change when there is work to prevent relapse and to sustain the gains made from the actions taken.

Theories of management style Traditional style

Manager's role in a strict hierarchy Focus on efficient and consistent job performance

PICC line

Measure 2 in above insertion site around arm when inserted for baseline and @ every assessment Mark with permanent marker where measured Document catheter length from insertion site to hub Change dressing and swab cap w/in 24 hours

Which of the following is the largest single source of reimbursement for home healthcare services? a) Patient's self-pay b) Medicaid c) Medicare d) Private insurance

Medicare Explanation: Medicare is the largest single source of reimbursement for home healthcare services. Other sources of reimbursement may include Medicaid, private insurance, self-pay, and other public funding

The nurse manager notices a trend in nurses routinely arriving to work late. How can the nurse use the transactional leadership style to get nurses to arrive to work on time? a) Monitor time clock reports on a monthly basis and enter all nurses who have not been tardy into a raffle for a paid day vacation. b) Generate a report of nurses who routinely arrive to work late and set an example by firing the nurse who has been tardy the most. c) Allow the nursing staff to develop a plan to decrease tardiness within the unit. d) Emphasize the importance of arriving to work on time during a monthly unit meeting.

Monitor time clock reports on a monthly basis and enter all nurses who have not been tardy into a raffle for a paid day vacation. Correct Explanation: Transactional leadership style involves creating a reward system to get the desired outcome. Entering the nurses who have not been tardy is the only option that provides a reward. Allowing the nursing staff to assist in correcting the problem is an example of democratic leadership style. Emphasizing the importance of arriving to work on time is an example of autocratic leadership. Firing is an example of coercive power.

Which of the following nursing interventions must a nurse perform when administering prescribed vasopressors to a patient with a cardiac dysrhythmia? a) Document heart rate before and after administration b) Monitor vital signs and cardiac rhythm c) Keep the patient flat for one hour after administration d) Administer every five minutes during cardiac resuscitation

Monitor vital signs and cardiac rhythm Correct Explanation: The nurse should monitor the patient's vital signs and cardiac rhythm for effectiveness of the medication and for side effects and should always have emergency life support equipment available when caring for an acutely ill patient. The side effects of vasopressor drugs are hypertension, dysrhythmias, pallor, and oliguria. It is not necessary to place a patient flat during or after vasopressor administration. When administering cholinergic antagonists, documentation of the heart rate is necessary.

Which of the following nursing interventions must a nurse perform when administering prescribed vasopressors to a patient with a cardiac dysrhythmia? a) Monitor vital signs and cardiac rhythm b) Keep the patient flat for one hour after administration c) Administer every five minutes during cardiac resuscitation d) Document heart rate before and after administration

Monitor vital signs and cardiac rhythm Explanation: The nurse should monitor the patient's vital signs and cardiac rhythm for effectiveness of the medication and for side effects and should always have emergency life support equipment available when caring for an acutely ill patient. The side effects of vasopressor drugs are hypertension, dysrhythmias, pallor, and oliguria. It is not necessary to place a patient flat during or after vasopressor administration. When administering cholinergic antagonists, documentation of the heart rate is necessary

The nurse is aware that a community health center provides which of the following? a) Community Health Centers serving the primary health care needs of more than a million patients in over 900 locations across the United States b) Ensuring that clients under 62 years of age have access to health care regardless of ability to pay c) Community health centers providing mostly obstetric and gynecological services (OB/GYN), regardless of the ability to pay d) Mostly primary care and education regionally for vulnerable geographic populations

Mostly primary care and education regionally for vulnerable geographic populations Explanation: These centers are mostly primary care and education centers for vulnerable populations. They are open to patients of any age and are not primarily OB/GYN centers. The centers provide care to 22 million clients in 9000 centers

What best describes the nurse's role in disaster preparedness? a) Administration of all of the medications b) Performance of all of the skills such as IV insertion and wound care c) Multiple roles including triage and the distribution of resources d) Counseling the victims and families

Multiple roles including triage and the distribution of resources Correct Explanation: Nurses will perform multiple roles when assisting with a disaster, including triage, procedures, counseling, and distribution of resources.

The nurse is analyzing a 6-second electrocardiogram (ECG) tracing. The P waves and QRS complexes are regular. The PR interval is 0.18 seconds long, and the QRS complexes are 0.08 seconds long. The heart rate is calculated at 70 bpm. The nurse correctly identifies this rhythm as which of the following? a) Junctional tachycardia b) Normal sinus rhythm c) Sinus tachycardia d) First-degree atrioventricular (AV) block

Normal sinus rhythm Explanation: The ECG tracing shows normal sinus rhythm (NSR). NSR has the following characteristics: ventricular and atrial rate: 60 to 100 beats per minute (bpm) in the adult; ventricular and atrial rhythm: regular; and QRS shape and duration: usually normal, but may be regularly abnormal; P wave: normal and consistent shape, always in front of the QRS; PR interval: consistent interval between 0.12 and 0.20 seconds and P:QRS ratio: 1:1.

Shared governance

Nurses participate in decision making at all levels of the organization Representatives of nurses on decision making boards or committees Employees will be more committed to an organization's goals if they have input Nurse manager is a consultant, teacher, collaborator

A nurse is caring for a client with cystic fibrosis. With which members of the health care team is it most appropriate for her to collaborate? Select all that apply. a) Physical therapy b) Nutritional services c) Social services d) Respiratory therapy e) Occupational therapy

Nutritional services • Physical therapy • Respiratory therapy Explanation: The major objectives of therapy for cystic fibrosis are promoting secretion clearance, controlling infection, and providing adequate nutrition. The respiratory therapist would help the client clear his secretions. Nutritional services are vital in promoting optimal nutrition. Exercise, a component of physical therapy, is important in clearing the airways. Social services and occupational therapy could play a role in this client's care but aren't as important as nutrition and physical therapy.

Which is a task that the nurse can delegate to the unlicensed assistive personnel (UAP)? a) Assisting a client with orthostatic hypotension to get out of bed b) Monitoring vital signs on a client with an active gastrointestinal bleed c) Teaching a client with diabetes how to inject insulin subcutaneously d) Obtaining intake and output on a client with a foley catheter

Obtaining intake and output on a client with a foley catheter Correct Explanation: The nurse should delegate tasks to the UAP that consider the scope of practice of the UAP, as well as the client?s stability and the complexity of the task. The client with a foley catheter who needs intake and output involves a routine process on a medical-surgical unit and would be within the scope of practice of the UAP. A client who is actively bleeding or has orthostatic hypotension is unstable, and these tasks should not be delegated to the UAP. Teaching a client about injection of insulin should not be delegated because it is a complex task that is outside the scope of practice for a UAP

Which action may a nurse on the orthopedic unit safely delegate to a licensed practical nurse (LPN)? a) Obtaining vital signs during blood administration b) Taking a telephone order for pain medications for a postoperative client c) Teaching a client receiving warfarin about follow-up care d) Assessing the hip wound during a dry sterile dressing change

Obtaining vital signs during blood administration Correct Explanation: The nurse may safely delegate obtaining vital signs during blood administration to the LPN. Teaching the client taking warfarin about follow-up care, assessing a hip wound, and taking a telephone order are actions that must be taken by the registered nurse because they aren't within the scope of LPN practice.

Which action may a nurse on the orthopedic unit safely delegate to a licensed practical nurse (LPN)? a) Obtaining vital signs during blood administration b) Assessing the hip wound during a dry sterile dressing change c) Teaching a client receiving warfarin about follow-up care d) Taking a telephone order for pain medications for a postoperative client

Obtaining vital signs during blood administration Explanation: The nurse may safely delegate obtaining vital signs during blood administration to the LPN. Teaching the client taking warfarin about follow-up care, assessing a hip wound, and taking a telephone order are actions that must be taken by the registered nurse because they aren't within the scope of LPN practice.

A nurse is researching health care in rural areas. Based on a 2013 U.S. Subcommittee on Primary Health and Aging, which of the following statements regarding access to primary health care is correct? a) Despite the fact that over half of patient visits are for primary care, only 17% of the nation?s medical school graduates now choose a primary care career. b) One in five Americans live in areas without adequate access to primary healthcare due to a shortage of primary care providers. c) Fewer than one in five physicians specialize in primary care. d) Over 100,000 people die each year because they do not have health insurance and do not get to a doctor on time.

One in five Americans live in areas without adequate access to primary healthcare due to a shortage of primary care providers. Explanation: It is estimated that about 45,000 people die each year in the United States due to access issues. Seven percent of the nation's medical school graduates choose a primary care career. Currently, one in three physicians specializes in primary care

The nurse is conducting a community education program on chronic illness. The nurse evaluates that additional education is needed when the participants make which of the following statements? a) Most people with chronic illness have health insurance. b) Chronic illness is increasing developing countries. c) Chronic illness is a leading cause of death. d) Out-of-pocket expenses for chronic illness are low.

Out-of-pocket expenses for chronic illness are low. Explanation: The nurse determines that additional education is needed when the participants state that out-of-pocket expenses for chronic illness are low. Although the majority of patients with chronic illness have health insurance, out of pocket expenses are high and are increasing. Chronic illness is a leading cause of death and is on the rise in developing countries.

Which of the following is an important role for a nurse in the health care delivery system? a) Balance of work with leisure activities b) Participation in disease prevention and health promotion activities c) Participation in treatment decisions regarding health restoration d) Participation in the diagnosis and treatment of the disease

Participation in disease prevention and health promotion activities Explanation: Nurses work in various settings, such as adhering to facility policies and state nurse practice acts. A nurse participates in disease prevention and health promotion activities for patients, family members, and communities. In the health care delivery system, balancing work with leisure activities is not the function of a nurse. A nurse does not participate in the diagnosis and treatment of a disease. A nurse also does not participate in treatment decisions regarding health restoration

A patient admitted to the telemetry unit has a serum potassium level of 6.6 mEq/L. Which of the following electrocardiographic (ECG) characteristics is commonly associated with this laboratory finding? a) Prolonged QT interval b) Flattened P waves c) Peaked T waves d) Occasional U waves

Peaked T waves Explanation: The patient's serum potassium level is high. The T wave is an ECG characteristic reflecting repolarization of the ventricles. It may become tall or "peaked" if a patient's serum potassium level is high. The U wave is an ECG waveform characteristic that may reflect Purkinje fiber repolarization. It is usually seen when a patient's serum potassium level is low. The P wave is an ECG characteristic reflecting conduction of an electrical impulse through the atria and is not affected by a patient's serum potassium level. The QT interval is an ECG characteristic reflecting the time from ventricular depolarization to repolarization, and is not affected by a patient's serum potassium level.

Which of the following situations is an example of battery that the nurse may experience while performing her nursing duties at the health care facility? a) Taking the client's photographs without consent b) Witnessing a procedure done on client without his consent c) Performing a surgical procedure without getting consent d) Telling the client that he cannot leave the hospital

Performing a surgical procedure without getting consent Correct Explanation: Performing a surgical procedure without the client's consent is an example of battery. To protect health care workers from being charged with battery, adult clients are asked to sign a general permission for care and treatment during admission and additional written consent forms for tests, procedures, or surgery. Telling the client not to leave the hospital is a false imprisonment. Taking the client's photographs without his permission and witnessing a procedure done on him without consent is violation of the client's privacy.

The nursing supervisor visits the emergency department and informs the department manager that tornado victims are expected to arrive within the hour. The department manager indicates the department has been slow and requests information regarding possible numbers of victims. The department manager reports supplies were just fully stocked but two nurses are ill with influenza and were unable to report for their shift. Which resource does the department manager need to organize to respond to the disaster? a) Environment b) Equipment c) Clients d) Personnel

Personnel Correct Explanation: A sufficient number of nurses are needed to respond to the disaster. The department is not full of clients and sufficient supplies are available.

Other Interruptions

Phone calls, text messages Set a specific time for them E-mail Check only at specific times Drop-in visitors Control frequency and duration Meetings Schedule only as necessary, have an agenda, and set time limit

Removing tube

Place petroleum guaze, will need supplemental O2 Assess respiratory status (color, effort, O2 sats) then equiptment

Preceptor/Mentor

Preceptor--an experienced nurse who provides support and is a clinical role model for the new nurse Preceptor is assigned to assist in improving clinical nursing skill and judgment necessary for effective practice in their environment Mentor is an informal position who may assist a nurse in career progression, role assimilation, or other aspects of career or role performance.

Which type of agencies provide documented home health care services? a) Private businesses b) Off-duty neighborhood nurses c) Unofficial neighborhood groups d) Hospital-based agencies e) Private not-for-profit agencies

Private businesses • Private not-for-profit agencies • Hospital-based agencies Correct Explanation: Home health care services are provided by official, publicly funded agencies; nonprofit agencies; private businesses; private not-for-profit agencies; and hospital-based agencies. Unofficial neighborhood groups and off-duty neighborhood nurses are not agencies that provide documented home health services

Nursing Interventions

Proper hand hygiene prior to handling CVC Sterile technique with all dressing changes Change of external hubs or caps with dressing changes and use biopatch Scrub the hub for 20-30 seconds with alcohol Flush CVC with 10 ml NS before and after use, Heparin per protocol and device

Performance improvement is an important component of continuous quality improvement. Which action should an effective nurse-manager take when conducting performance evaluations? a) Conduct performance evaluations in a group setting so input from peers and subordinates influences evaluation of a staff member's effectiveness. b) Whenever possible, delegate responsibility for conducting performance evaluations to primary nurses to help them achieve professional growth. c) Provide feedback on strengths as well as areas for improvement and clarify what she expects the staff member to accomplish before the next performance evaluation. d) Provide written documentation of areas for improvement. Areas of strength needn't be documented because these areas are complimentary and don't describe actions the staff member must take to improve.

Provide feedback on strengths as well as areas for improvement and clarify what she expects the staff member to accomplish before the next performance evaluation. Correct Explanation: An effective performance evaluation recognizes strengths, identifies areas for improvement, and clarifies performance expectations. The nurse-manager should conduct performance evaluations privately, not in front of others. She should document in writing all components of a performance evaluation. Although input from staff members can be useful in preparing performance evaluations, delegating all responsibility to others is inappropriate. The nurse-manager is responsible for the performance of the staff.

Megan is the nurse manager for a surgical unit and is planning a significant change in how the unit functions. She plans to include some things that will help to overcome any resistance to the change from the staff. Which of the following might help overcome resistance to Megan's planned change? a) Refuse to hear any discussion b) Demand loyalty and commitment c) Threaten to end staff's employment d) Provide incentives, such as time off

Provide incentives, such as time off Correct Explanation: Nurses acting as change agents would it beneficial to: (1) explain the proposed change to all affected people in simple, concise language; (2) list the advantages of the proposed change, both for the individual and for members of the group; (3) relate the proposed change to the person's or group's existing beliefs and values; (4) help overcome resistance by providing opportunities for open communication and feedback; (5) indicate clearly how the change will be evaluated; (6) introduce change gradually and involve everyone affected by the change in the design and implementation of the process; (7) provide incentives for commitment to change, such as money, status, time off, or a better working environment. Threatening to end employment, demands for loyalty, and refusal to listen to discussion are sure ways to turn staff against the planned change.

In preparation for discharge, the nurse is reviewing information related to new dietary guidelines with the client. This is an example of which step in discharge planning? a) Providing client teaching. b) Assessing the client's needs and identifying problems. c) Developing goals with the client. d) Making home healthcare referrals.

Providing client teaching. Explanation: The nurse is teaching the client important information about self-care at home prior to discharge. The initial step in discharge planning is collecting and organizing data about the client, because this provides information on the client's healthcare needs. Home referrals may be made after the teaching process, based upon orders provided by the physician. Developing goals may occur after the teaching process, because the goals need to be realistic.

What nursing function would be most commonly found in an ambulatory care facility? a) Serving as an administrator or manager b) Assessing the home environment c) Providing direct patient care d) Educating individuals or groups

Providing direct patient care Correct Explanation: Ambulatory care centers and clinics (agencies that deliver outpatient medical care) may be located in hospitals, may be a freestanding service provided by a group of health care providers who work together, or may be managed by an APRN

A client admitted with bacterial meningitis must be transported to the radiology department for a repeat computed tomography scan of the head. His level of consciousness is decreased, and he requires nasopharyngeal suctioning before transport. Which infection control measures are best when caring for this client? a) Put on gloves, a mask, and eye protection. b) Take no special precautions for this client. c) Use standard precautions, which require gloves for suctioning. d) Put on gloves, a mask, and eye protection during suctioning, and then apply a mask to the client's face for transport.

Put on gloves, a mask, and eye protection during suctioning, and then apply a mask to the client's face for transport. Correct Explanation: Bacterial meningitis is spread through contact with infected droplets. The nurse should wear gloves, a mask, and eye protection when suctioning the client. Additionally, the client should wear a mask when out of the isolation room for diagnostic testing. Standard precautions don't adequately protect staff and other clients from bacterial meningitis. (

Which of the following ECG waveforms characterizes conduction of an electrical impulse through the left ventricle? a) QRS complex b) PR interval c) QT interval d) P wave

QRS complex Correct Explanation: The QRS complex represents ventricular depolarization. The P wave is an ECG characteristic reflecting conduction of an electrical impulse through the atria. The PR interval is a component of an ECG tracing reflecting conduction of an electrical impulse through the AV node. The QT interval is an ECG characteristic reflecting the time from ventricu

A nurse-manager at a community hospital is discussing the concept of Pay for Performance with the medical-surgical unit's physicians, nurses, and other members of the health care team. Which of the following statements made by the nurse-manager is consistent with this concept? a) Financial incentives are employed to reward providers for the achievement of a range of patient objectives. b) Pay for Performance is to be instituted primarily in small community hospitals with fewer than 100 beds, as patient outcomes are worse there. c) Quality of care is being measured and used to evaluate hospitals and other providers and to award reimbursement. d) Pay for Performance includes only demonstrated delivery efficiencies and improved quality and patient safety.

Quality of care is being measured and used to evaluate hospitals and other providers and to award reimbursement. Explanation: Financial incentives are provided for the achievement of the payer, not patient objectives, and includes not only delivery efficiencies, improved quality, and patient safety but also submission of data to the payer. Pay for Performance is not limited to any particular hospital size

A nurse-manager at a community hospital is discussing the concept of Pay for Performance with the medical-surgical unit's physicians, nurses, and other members of the health care team. Which of the following statements made by the nurse-manager is consistent with this concept? a) Pay for Performance is to be instituted primarily in small community hospitals with fewer than 100 beds, as patient outcomes are worse there. b) Financial incentives are employed to reward providers for the achievement of a range of patient objectives. c) Pay for Performance includes only demonstrated delivery efficiencies and improved quality and patient safety. d) Quality of care is being measured and used to evaluate hospitals and other providers and to award reimbursement.

Quality of care is being measured and used to evaluate hospitals and other providers and to award reimbursement. Explanation: Financial incentives are provided for the achievement of the payer, not patient objectives, and includes not only delivery efficiencies, improved quality, and patient safety but also submission of data to the payer. Pay for Performance is not limited to any particular hospital size.

A nurse-manager works for a nonprofit health care corporation whose revenues have significantly exceeded annual expenses. The nurse-manager has been told to anticipate which action? a) Identifying revenue as profit b) Receiving a portion of the revenue to improve client services on the unit c) Reducing operating expenses to help the organization pay taxes on the revenue d) Dividing revenue among stockholders as dividends

Receiving a portion of the revenue to improve client services on the unit Correct Explanation: In a nonprofit organization, revenue exceeding expenses is tax-exempt and is usually reinvested in the organization and used to improve services. A for-profit organization calls revenue in excess of expenses a profit and divides it as a dividend among stockholders or reinvests it in the organization

Transformational leadership

Reconsiders the characteristics of the leader-manager Emphasizes the vision that the leader-manager shares with the group Stresses the importance of preparing people for change Four primary factors Charisma Inspirational motivation Intellectual stimulation Contingent reward

An operating room (OR) nurse on the facility's infection control team notices that a coworker in the OR is wearing artificial nails. Which of the following is the appropriate action/response by the nurse? a) Remind coworker of the need to wear gloves b) No action is needed at this time c) Remind coworker that artificial nails increase infections d) Remind coworker to wash hands for 2 minutes

Remind coworker that artificial nails increase infections Correct Explanation: Artifical nails are associated with higher bacterial counts and therefore increase the client's risk for infection. Washing hands and wearing gloves do not make wearing artificial nails appropriate. In fact, wearing artificial nails in the OR is a citable offense during the Joint Commission accreditation process.

A registered nurse (RN) and licensed practical nurse (LPN) are administering medications on the neurologic floor. The LPN prepares to administer phenytoin to a client with a history of seizures. As the LPN walks into the room, she hands the medication to a nursing assistant. The LPN asks the nursing assistant to give the client the medication after completing the client's morning care. What should the registered nurse do? a) Allow the nursing assistant to administer this dose and tell the LPN later that it's her responsibility to administer the medication. b) Take the medication from the nursing assistant and administer it. c) Remind the LPN that she must administer the medications herself. d) Do nothing because the client has been taking the medication for a long time.

Remind the LPN that she must administer the medications herself. Correct Explanation: The RN should intervene immediately by reminding the LPN that it's her responsibility to administer the medications. The RN should reinforce to the LPN that medication administration is beyond the scope of practice for a nursing assistant, and that allowing the nursing assistant to administer medications could lead to client injury. Although the client has been taking the medication for a long time, the responsibility for medication administration lies with the RN and LPN, not the nursing assistant. It's important for the nurse to intervene at the time of the incident to prevent injury. The registered nurse shouldn't administer the medication because she didn't prepare the medication for administration herself.

An on-duty nurse discovers that her colleague is pilfering medicines. According to the Nurse Practice Acts, what should the nurse do? a) Keep silent and overlook the incident b) Inform the local police station c) Discuss this incident with the colleague d) Report the incident to the supervisor

Report the incident to the supervisor Correct Explanation: According to the Nurse Practice Acts, the nurse should report the incident to the supervisor. Laws are enacted to regulate the practice of nursing and may be used to decide upon an appropriate action. Discussing the incident with a colleague may alarm the colleague who is pilfering medicines, who may then become cautious. The nurse should not overlook the incident because pilfering of medicines is an offense. Calling local police may lead to undue interference.

During the admission assessment of a 40-year-old female patient with a suspected mandibular fracture, the patient discloses to the nurse that her injury came as a result of her husband hitting her. Which of the following actions should the nurse prioritize when responding to this disclosure? a) Informing the patient of her right to keep this information private b) Performing an assessment to confirm the patient's statement c) Reporting the abuse to the appropriate authorities d) Ensuring the patient's statement is confirmed by another nurse

Reporting the abuse to the appropriate authorities Correct Explanation: Nurses have a legal and ethical obligation to report cases of abuse. It would be inappropriate and likely unethical to require a third party witness to the statement or to withhold action pending assessment results. The nurse's obligation to report abuse legally supersedes the patient's right to privacy

When assessing the older adult, the nurse anticipates an increase in which component of the respiratory status? a) Residual lung volume b) Cough efficiency c) Vital capacity d) Gas exchange and diffusing capacity

Residual lung volume Correct Explanation: With an increase in residual lung volume the patient experiences fatigue and breathlessness with sustained activity. The nurse anticipates decreased vital capacity. The nurse anticipates decreased gas exchange and diffusing capacity resulting in impaired healing of tissues due to decreased oxygenation. The nurse anticipates difficulty coughing up secretions due to decreased cough efficiency.

A home care nurse has just received a referral for home care services. Which of the following should the nurse do first? a) Contact the referring agency to confirm receipt of the referral. b) Review the referral form for the pertinent information. c) Check the home care agency's policies about how to make a visit. d) Telephone the client to determine a visit schedule.

Review the referral form for the pertinent information. Correct Explanation: Upon receipt of a referral for home care, the nurse should review the referral form and other pertinent data about the client. The nurse would need not contact the referring agency unless the referral is unclear or if important information is missing. Once the nurse had the necessary information, he or she would telephone the client to schedule the first visit. The nurse should have a clear understanding of the home care agency's policies and procedures for a visit before accepting any referral for a client.

An acute care facility follows the unit dose supply method to supply medication to the clients. What is meant by the unit dose supply method? a) Self-contained packets that hold one tablet or capsule for individual clients b) Systems that contain frequently used medication for that unit c) A container with enough prescribed medications for several days for a client d) A supply that remains on the nursing unit for use in emergency

Self-contained packets that hold one tablet or capsule for individual clients Correct Explanation: The nurse should understand that a unit dose supply method is a method in which self-contained packets hold one tablet or capsule for an individual client. An individual supply is a container with enough of the prescribed medication for several days or weeks and is common in long-term care facilities such as nursing homes. A stock supply remains on the nursing unit for use in an emergency or so that a nurse can give a medication without delay. Some facilities use automated medication-dispensing systems, which contain frequently used medications for that unit, any as-needed (PRN) medications, controlled medications, and emergency medications.

Which of the following dysrhythmias are common in older patients? a) Sinus bradycardia b) Atrial fibrillation c) Sinus tachycardia d) Ventricular tachycardia

Sinus bradycardia Correct Explanation: Sinus bradycardia is a common dysrhythmia in older patients. Sinus tachycardia, atrial fibrillation, and ventricular tachycardia are not common dysrhythmias in older patients.

A 66-year-old female client is having cardiac diagnostic tests to determine the cause of her symptoms. In her follow-up visit to the cardiologist, she is told that she has a dysrhythmia at a rate slower than 60 beats/minute. What type of dysrhythmia did the tests reveal? a) Heart block b) Sinus bradycardia c) None d) Atrial bradycardia

Sinus bradycardia Correct Explanation: Sinus bradycardia is a dysrhythmia that proceeds normally through the conduction pathway but at a slower than usual (≤60 beats/minute) rate. Sinus bradycardia is a slower than usual (≤60 beats/minute) heart rate

A hospital social worker speaks to a group of students orienting to a local hospital for their first medical surgical nursing rotation. Which of the following roles is within the social worker's scope of practice? Select all that apply. a) Social workers assist patients and families in dealing with the social, emotional, and environmental factors that affect their well-being b) Social workers assist with discharge planning c) Social workers provide direct, in-hospital, psychological counseling d) Social workers make referrals to appropriate community resources e) Social workers provide assistance with securing medical devices and supplies

Social workers assist patients and families in dealing with the social, emotional, and environmental factors that affect their well-being • Social workers make referrals to appropriate community resources • Social workers provide assistance with securing medical devices and supplies • Social workers assist with discharge planning Explanation: Hospital based social workers do not generally provide direct counseling for psychiatric patients, or other patients with psychiatric disorders. All other choices are correct

Time Wasters

Staff interruptions Meetings without a clear purpose Goals, objectives, and priorities that are not measurable Plans without time parameters Disorganized files or papers Time logs, not analyzed Tasks/activities that can be delegated Waiting for others Inability to say no

Plan

Start with client/family preferred outcomes in mind Refer to NCBON NPA and statutes, organizational job descriptions, policies, roles Students, novices, floats, need variable levels of supervision, guidance, support RN is accountable for nursing judgment decisions and supervision of delegation Cannot delegate nursing function to a non-nurse Clinical situations are dynamic, not static

Nurse Practice Acts are examples of which type of laws? a) Constitutional laws b) Common law c) Administrative law d) Statutory laws

Statutory laws Correct Explanation: Nurse Practice Acts are statutory laws. Statutory laws must be in keeping with both the federal constitution and the state constitution.

A group of students are reviewing information on the roles of a nurse. The students demonstrate understanding of the information when they identify which activity as associated with the nurse's role as a leader? a) Supervision of unlicensed assistive personnel b) Client education about a disease condition c) Identification of a problem related to patient care d) Development of a plan of care for a client

Supervision of unlicensed assistive personnel Correct Explanation: The leadership role involves those actions that nurses execute when they assume responsibility for the actions of others directed toward determining and achieving client goals, for example supervising unlicensed assistive personnel. Identification of a problem related to patient care implies the researcher role where this problem would serve as the basis for the identification of researchable questions. Client education and development of a care plan would be associated with the practitioner role.

Theories of management style

Systems theory Approaches problems by looking at inputs, transformation processes, outputs, and feedback Contingency theory Blends concepts of traditional, behavioral, and systems theory Determine which method based on the situation

Which task should a nurse choose to delegate to a nursing assistant? Select all that apply. a) Performing a blood glucose check b) Taking a client's vital signs c) Documenting a client's oral intake d) Evaluating a client's response to a blood pressure medication e) Assessing a client's pain

Taking a client's vital signs • Performing a blood glucose check • Documenting a client's oral intake Correct Explanation: Registered nurses are responsible for all phases of the nursing process. These responsibilities include assessing a client's pain and evaluating a client's response to treatment. A nurse may delegate tasks such as taking vital signs, documenting intake and output, and performing blood glucose checks if she follows the five rights of delegation. The five rights of delegation include: right task (the task is within the delegate's scope of practice), right person (the person is competent to perform the task), right communication (the nurse gives the right directions to complete the task), right feedback (the nurse works collaboratively with the delegate), and right follow-up (the nurse follows-up on the task after it has been completed).

A nurse-manager on an oncology unit has been informed that she must determine which nursing care delivery system (NCDS)/nursing care delivery model (NCDM) is best for efficient client care, client satisfaction, and cost reduction. Knowing that two or three registered nurses, four licensed practical nurses, and five nursing assistants are generally on duty on each shift and that the clients can easily be grouped by geographic location and client care needs, the nurse-manager and her staff appropriately decide to implement which NCDS/NCDM? a) Primary nursing b) Case management c) Team nursing d) Functional nursing

Team nursing Correct Explanation: Team nursing is efficient and less costly to implement than primary or case management systems. Because staff members know each other well, they can function effectively as a team. Although functional nursing is the most cost-effective, care is commonly fragmented and clients are less satisfied. Case management and primary nursing require more registered nurses than are available.

A registered nurse has had her license suspended after being convicted of being impaired at work. What governing body has the authority to revoke or suspend a nurse's license? a) The National League for Nursing b) The employing healthcare institution c) The Supreme Court d) The State Board of Nurse Examiners

The State Board of Nurse Examiners Correct Explanation: The State Board of Nurse Examiners in the United States may revoke or suspend a nurse's license or registration for drug or alcohol abuse, which is the most common reason.

A registered nurse has had her license suspended after being convicted of being impaired at work. What governing body has the authority to revoke or suspend a nurse's license? a) The State Board of Nurse Examiners b) The employing healthcare institution c) The Supreme Court d) The National League for Nursing

The State Board of Nurse Examiners Correct Explanation: The State Board of Nurse Examiners in the United States may revoke or suspend a nurse's license or registration for drug or alcohol abuse, which is the most common reason.

A nursing student working today on the cardiac unit asks the instructor why loss of the "atrial kick" causes a decrease in cardiac output, because ventricles are still contracting. The instructor's best answer is which of the following? a) "The atrial contraction fills the ventricles and accounts for nearly one third of the volume ejected during ventricular contraction." b) "The atrial kick is only helpful if the ventricles are failing." c) "The atrial kick is never lost; there is always some atrial kick present." d) "Loss of the atrial kick does not affect cardiac output; cardiac output remains the same, with or without it."

The atrial contraction fills the ventricles and accounts for nearly one third of the volume ejected during ventricular contraction." Correct Explanation: The atria contract slightly before the ventricles, giving the atria time to contract and fill the ventricles with blood. This part of atrial contraction is often referred to as the "atrial kick" and accounts for nearly one third of the blood volume ejected during ventricular contraction. The "atrial kick" is lost with an atrial fibrillation rhythm

The registered nurse is working with an unlicensed assistive personnel. Which client should the nurse not delegate to the unlicensed assistive personnel? a) The client who is pleasantly confused and requires assistance to the bathroom. b) The client who requires assistance dressing in preparation for discharge. c) The client with continuous pulse oximetry who requires pharyngeal suctioning. d) The client who needs vital signs taken following infusion of packed red blood cells.

The client with continuous pulse oximetry who requires pharyngeal suctioning. Correct Explanation: The nurse needs to perform the pharyngeal suctioning of the client with continuous pulse oximetry. This client requires the nurse to evaluate the client's response in pulse oximetry to the suctioning. The nurse can delegate the other clients to the unlicensed assistive personnel.

Which of the following aspects of nursing would be most likely defined by legislation at a state level? a) The criteria that a nurse must consider when delegating tasks to unlicensed care providers. b) The process that nurses must follow when handling and administering medications. c) The criteria that patients must meet in order to qualify for Medicare or Medicaid. d) The differences in the scope of practice between registered nurses (RNs) and licensed practical nurses (LPNs).

The differences in the scope of practice between registered nurses (RNs) and licensed practical nurses (LPNs). Correct Explanation: The scope of practice defines the parameters within which nurses provide care and is established by state legislation, most commonly in the form of a Nurse Practice Act. The criteria and due process for delegation in the clinical setting is addressed by a state board of nursing. Qualification criteria for programs such as Medicare and Medicaid are established by federal legislation while the process for safe and appropriate medication administration is defined and monitored by a state board of nursing

Nurses now have the option to practice in a variety of settings. One of the fastest growing venues of practice for the nurse in today's health care climate is home health care. What is the basis for the growth in this health care setting? a) The chronic nursing shortage b) The discharge home of patients who are more critically ill. c) The preference of nurses to work during the day instead of evening or night shifts d) The focus on treatment of disease

The discharge home of patients who are more critically ill. Correct Explanation: With shorter hospital stays and increased use of outpatient health care services, more nursing care is provided in the home and community setting. The other answers are incorrect because they are not the basis for the growth in nursing care delivered in the home setting

Which nurse is using criteria to determine expected standards of performance? a) The new graduate nurse consults the policies and procedures of the institution prior to skill implementation. b) The nurse preceptor provides feedback to the new graduate nurse after six weeks of orientation. c) The nurse manager provides the staff nurse feedback regarding job performance for the previous year. d) The nurse seeks information from the unlicensed nursing personnel regarding the family's response to the nurse's teaching

The new graduate nurse consults the policies and procedures of the institution prior to skill implementation. Explanation: Standards are the levels of performance accepted and expected by the nursing staff and other health team members, such as institutional policies and procedures. The nurse preceptor providing feedback to the new graduate nurse after six weeks of orientation is an example of peer review. The nurse manager providing the staff nurse feedback regarding job performance for the previous year is typical of an annual employee review

Which of the following is an area of potential liability for the nurse? Select all that apply. a) The nurse fails to document refusal by the client to ambulate following surgery. b) The nurse administers the client's preoperative medication after the informed consent is signed. c) The nurse documents that the client's blood pressure has increased from 118/72 to 188/98 and decides to re-take the blood pressure in an hour. d) The nurse documents that the client accurately prepared the correct amount of insulin after instruction was given. e) The nurse notifies the physician of the client's adverse reaction to a medication.

The nurse fails to document refusal by the client to ambulate following surgery. • The nurse documents that the client's blood pressure has increased from 118/72 to 188/98 and decides to re-take the blood pressure in an hour. Explanation: Areas of potential liability would include failure to document refusal by the client to participate in the treatment regimen, such as ambulation after surgery, and failure to assess the client in a timely manner. Waiting an hour to reassess a significant elevation in blood pressure does not meet the standard of care. Reporting a client's adverse reaction to a medication, administering preoperative medication after the informed consent is signed, and documenting the client's response to teaching are nursing behaviors that meet the standard of care.

The nurse is conducting a peer review of a nursing colleague. Which action by the nurse is an example of peer review? a) The new graduate nurse consults the policies and procedures of the institution prior to skill implementation. b) The nurse preceptor provides feedback to the new graduate nurse after six weeks of orientation. c) The nurse seeks feedback from the nurse manager regarding job performance for the previous year. d) The nurse seeks information from the unlicensed nursing personnel regarding the family's response to the nurse's teaching.

The nurse preceptor provides feedback to the new graduate nurse after six weeks of orientation. Correct Explanation: Peer review involves the evaluation of one staff member by another staff member on the same level in the hierarchy of the organization for the purpose of professional performance improvement. The nurse preceptor providing feedback to the new graduate nurse after six weeks of orientation is an example of peer review.

A physician is called to see a client with angina. During the visit the physician advises the nurse to decrease the atenolol to 12.5 mg. However, since the physician is late for another visit, she requests that the nurse write down the order for her. What should be the appropriate nursing action in this situation? a) The nurse should write the order and implement it. b) The nurse should inform the client of the change in medication. c) The nurse should remind the physician later to write the work order. d) The nurse should ask the physician to come back and write the order.

The nurse should ask the physician to come back and write the order. Correct Explanation: The nurse should ask the physician to come back later and write down the order. However, nurses are discouraged from following any verbal orders, except in emergency. The nurse should never write an order on a physician's behalf because this is a wrong practice. The client should be informed about the change of medications, but this is not an appropriate action. The nurse should not leave the work for a later time, because the nurse may forget it.

The nurse-manager of a 20-bed coronary care unit is not on duty when a staff nurse makes a serious medication error that results in a client's overdose. The client nearly dies. Which statement accurately reflects the accountability of the nurse-manager? a) Because the nurse-manager is off duty and not accountable for incidents that occur in his/her absence, he/she need not be notified. b) The nurse-manager only needs to be informed of the incident when he/she reports to work on the next scheduled day. c) The nurse-manager would receive a call at home from the on-duty nursing supervisor, apprising him/her of the problem as soon as possible. d) Although the nurse-manager is off duty and not responsible for what happened, the nursing supervisor would call the nurse-manager only if time permits.

The nurse-manager would receive a call at home from the on-duty nursing supervisor, apprising him/her of the problem as soon as possible. Correct Explanation: The nurse-manager is accountable for what happens on the unit 24 hours per day, 7 days per week. If a serious problem occurs, the nurse-manager should be notified as soon as possible. None of the other choices accurately reflect the nurse--manager's accountability in this situation

The licensed practical nurse is co-assigned with a registered nurse in the care of a client admitted to the cardiac unit with chest pain. The licensed practical nurse is assessing the accuracy of the cardiac monitor, which notes a heart rate of 34 beats/minute. The client appears anxious and states not feeling well. The licensed practical nurse confirms the monitor reading. When consulting with the registered nurse, which of the following is anticipated? a) The registered nurse stating to administer Lanoxin (digoxin) b) The registered nurse administering atropine sulfate intravenously c) The registered nurse stating to hold all medication until the pulse rate returns to 60 beats/minute d) The registered nurse stating to administer all medications accept those which are cardiotonics

The registered nurse administering atropine sulfate intravenously Correct Explanation: The licensed practical nurse and registered nurse both identify that client's bradycardia. Atropine sulfate, a cholinergic blocking agent, is given intravenously (IV) to increase a dangerously slow heart rate. Lanoxin is not administered when the pulse rate falls under 60 beats/minute. It is dangerous to wait until the pulse rate increases without nursing intervention or administering additional medications until the imminent concern is addressed

Your client has just been diagnosed with a dysrhythmia. The client asks you to explain normal sinus rhythm. What would you explain is the characteristic of normal sinus rhythm? a) The sinoatrial (SA) node initiates the impulse. b) Impulse travels to the atrioventricular (AV) node in 0.15 to 0.5 second. c) Heart rate between 60 and 150 beats/minute. d) The ventricles depolarize in 0.5 second or less.

The sinoatrial (SA) node initiates the impulse. Correct Explanation: The characteristics of normal sinus rhythm are heart rate between 60 and 100 beats/minute, the SA node initiates the impulse, the impulse travels to the AV node in 0.12 to 0.2 second, the ventricles depolarize in 0.12 seconds or less, and each impulse occurs regularly

The registered nurse (RN) wants to delegate measuring a client's urinary output to an unlicensed assistive personnel (UAP). Which factors should the nurse consider before delegating the task? a) Predictability of the UAP, the amount of time required for the task, and RN?s skill level b) The complexity of the activity, age of the UAP, and predictability of the outcome c) The context of the other patient needs, the desired outcome, and autonomy of the patient d) The stability of the patient?s condition, potential for harm, and complexity of the activity

The stability of the patient?s condition, potential for harm, and complexity of the activity Correct Explanation: RNs should consider the following when delegating tasks to UAPs: qualifications and capabilities of the UAP, stability of the patient?s condition, complexity of the activity to be delegated, potential for harm, predictability of the outcome, and overall context of other patient needs

A nurse is applying for professional liability insurance. The nurse knows that professional liability insurance is important for which of the following reasons? a) To obtain free medication for family b) To obtain sound compensation c) To be familiar with legal mechanisms d) To upgrade professional knowledge

To be familiar with legal mechanisms Explanation: The number of lawsuits involving nurses is increasing. It is to every nurse's advantage to obtain liability insurance and to become familiar with legal mechanisms, such as Good Samaritan laws and statutes of limitations that may prevent or relieve culpability and provide a sound legal defense. Professional liability insurance does not focus on the upgradation of nurse's professional knowledge, obtain free medication for family, or obtain sound compensation to a nurse.

A nurse is caring for a client who has undergone coronary angioplasty. The cardiac monitor is showing abnormal ECG waves, indicating arterial fibrillation. The nurse does not recognize the importance of the sign; as a result, the client's condition deteriorates and the client has to be taken up for an emergency procedure. Which of the following describes the nurse's legal liability? a) Tort b) Felony c) Slander d) Defamation

Tort Correct Explanation: A tort is a cause of action in which one person asserts that a physical, emotional, or financial injury was a consequence of another person's actions or failure to act. A tort implies that a person breached his or her duty to another person. In this case, the nurse had a duty that was breached. A felony is a serious criminal offense, such as murder. Defamation is an act in which untrue information harms a person's reputation. Slander is a character attack uttered orally in the presence of others.

Transactional leadership

Traditional manager focused on the day-to-day tasks of achieving organizational goals Relationship is based on exchange for some resource valued by the follower

Your patient is experiencing asymptomatic sinus tachycardia with a rate of 118. The nurse understands that the treatment of this condition includes: a) Administration of amiodarone b) Immediate defibrillation c) Electrical cardioversion d) Treating the underlying cause

Treating the underlying cause Correct Explanation: Sinus tachycardia occurs in response to an underlying condition and will usually resolve once that condition is corrected.

The nurse is participating in the care of a client requiring emergent defibrillation. The nurse will complete the following steps in which order? a) Apply the multifunction conductor pads to the patient's chest. b) Turn on the defibrillator and place it in "not sync" mode. c) Call "clear" three times ensuring patient and environmental safety. d) Charge the defibrillator to the prescribed voltage. e) Deliver the prescribed electrical charge.

Turn on the defibrillator and place it in "not sync" mode. • Charge the defibrillator to the prescribed voltage. • Apply the multifunction conductor pads to the patient's chest. • Call "clear" three times ensuring patient and environmental safety. • Deliver the prescribed electrical charge. Explanation: This is the sequence of events the nurse should implement when delivering emergent defibrillation. If not followed correctly, the patient and health care team may be placed in danger.

The nurse assigns an unlicensed assistive personnel (UAP) to care for a client who has a newly applied long-leg plaster cast. What should the nurse tell the UAP about proper care of the cast while it is drying? a) "Use a blow dryer on the cast for 15 minutes every 2 hours until the cast is dry." b) "Keep the cast covered with a sheet to protect it while drying." c) "Turn the client every 2 hours to promote even drying of the cast." d) "Carefully use your fingers to lift the cast and reposition the legs."

Turn the client every 2 hours to promote even drying of the cast." Correct Explanation: The client should be repositioned every 2 hours to promote even drying of the cast. The cast should be kept uncovered while drying to allow air to circulate around the cast and prevent heat from building up within it. It takes 24 to 72 hours for a plaster cast to dry; using a blow dryer may cause a heat burn and does not reduce the time for the cast to dry. The palms of the hands, not the fingers, should be used to move a drying cast in order to prevent indentations that can cause pressure points to develop.

The client receives a medication and develops a rash on the trunk of the body, itching, and dyspnea. This is an example of a: a) Time-out event. b) Sentinel event. c) Variance. d) Never event.

Variance. Explanation: This is an example of a variance. A variance, occurrence, or incident is a report completed when anything out of the ordinary results in or has the potential to result in harm to a client, employee, or visitor. A sentinel event is an unexpected occurrence involving death or serious physical or psychological injury, or the risk thereof. A never event is an extremely rare medical error that should never occur. A "time-out" procedure is done to correctly identify the client, the correct surgical site, and a determination that the client has provided written agreement on procedure to be performed by signing an informed consent

The nurse is working on a monitored unit assessing the cardiac monitor rhythms. Which waveform pattern needs attention first? a) Sustained asystole b) Ventricular fibrillation c) Supraventricular tachycardia d) Atrial fibrillation

Ventricular fibrillation Explanation: Ventricular fibrillation is called the rhythm of a dying heart. It is the rhythm that needs attention first because there is no cardiac output, and it is an indication for CPR and immediate defibrillation. Sustained asystole either is from death, or the client is off of the cardiac monitor. Supraventricular tachycardia and atrial fibrillation is monitored and reported to the physician but is not addressed first.

Dressing change

Wear mask and have pt wear mask or turn head if unable to

The nurse reports a nursing colleague on the unit who is lethargic and verbally responds in a slow manner. This is an example of: a) Ensuring adequate staffing. b) Delegating nursing care. c) Whistle-blowing. d) Collective bargaining

Whistle-blowing. Explanation: Whistle-blowing is when the nurse reports unsafe practice environments. Impaired nurses threaten the safety of clients in the clinical setting, as does inadequate staffing. Nurses may delegate or assign tasks involve in the delivery of nursing care to individuals as long as the individual has sufficient knowledge and skill to perform the assigned task. Collective bargaining is a legal process in which representatives of organized employees negotiate with employers about work conditions

The RN is working with hospital administration to transform care at their facility. Which of the following nursing competencies will be critical for the nurse to utilize? a) Work effectively in interdisciplinary teams. b) Navigate the electronic medical records system. c) Do things the way they have always been done. d) Correctly utilize and troubleshoot high-tech equipment.

Work effectively in interdisciplinary teams. Explanation: The RN working with administration to transform care will need to be able to work effectively as part of an interdisciplinary team. The nurse will need to work as a team member with members of administration, as well as representatives from other health disciplines involved in the project. The ability to use and troubleshoot equipment and to navigate the electronic medical records are important to the nurse, but will not necessarily help when working with administration to transform care. Doing things the way they have always been done is a barrier to transformation of care

Self-Discipline

Work from clearly defined priorities based on measurable, achievable objectives Establish realistic commitments to manage time effectively Do not procrastinate Handle each task once Communicate your own needs to others Learn to say "no" rather than over-extend commitments

A nurse-manager in the office of a group of surgeons has received complaints from discharged clients about inadequate instructions for performing home care. Knowing the importance of good, timely client education, the nurse-manager should take which steps? a) Because none of the clients suffered any serious damage, the nurse-manager can safely ignore their complaints. b) Inform the nurses who work in the facility that client education should be implemented as soon as the client is admitted to either the hospital or the outpatient surgical center. c) Work with the surgeons' staff and the nursing staff in the hospital and outpatient surgical center to evaluate current client education practices and make revisions as needed. d) Review and revise the way client education is conducted in the surgeons' office.

Work with the surgeons' staff and the nursing staff in the hospital and outpatient surgical center to evaluate current client education practices and make revisions as needed. Correct Explanation: Every nurse who provides client care should provide client education. Nurses must work together to establish the best methods of educating clients. The most appropriate response is to contact the facility's nurse-manager, not the nursing staff. Evaluating client education in only the surgeon's office doesn't consider the entire client education process and all of the staff providing it. Client education is an important nursing responsibility and every complaint deserves attention.

Theories of management style Behavioral viewpoint

Workers are motivated by social needs and get a sense of identity through their associations with one another Workers are more responsive to social forces exerted by their peers than to management's financial incentives and rules Workers respond to manager who can help satisfy their needs Managers need to coordinate the work of their subordinates democratically in order to improve efficiency

Thirty people are injured in a train derailment. Which client should be transported to the hospital first? a) an 80-year-old who has a compound fracture of the arm b) a 20-year-old who is unresponsive and has a high injury to his spinal cord c) a 25-year-old with a sucking chest wound d) a 10-year-old with a laceration on his leg

a 25-year-old with a sucking chest wound Correct Explanation: During a disaster, the nurse must make difficult decisions about which persons to treat first. The guidelines for triage offer general priorities for immediate, delayed, minimal, and expectant care. The client with a sucking chest wound needs immediate attention and will likely survive. The 80-year-old is classified as delayed; emergency response personnel can immobilize the fracture and cover the wound. The 10-year-old has minimal injuries and can wait to be treated. The client with a spinal cord injury is not likely to survive and should not be among the first to be transported to the health care facility

The charge nurse is making client care assignments for the evening shift. One of the licensed practical nurses (LPNs) is a new graduate in orientation. Which client would be an appropriate care assignment for this LPN? a) a 41-year-old client with unstable angina b) a 32-year-old client hospitalized for chemotherapy treatment c) a 72-year-old client with diverticulitis d) a 5-year-old client with Kawasaki's disease

a 72-year-old client with diverticulitis Correct Explanation: The client with diverticulitis will need care that the LPN should be able to provide safely. The client with angina is unstable and requires a registered nurse for continuous asses

A student nurse begins a clinical rotation in a long-term care facility and quickly realizes that certain residents have unmet needs. The student wants to advocate for these residents. Which statements reflect a correct understanding of advocacy? Select all that apply. A Advocacy is the protection and support of another's rights. B Patient advocacy is primarily done by nurses. C Patients with special advocacy needs include the very young and the elderly, those who are seriously ill, and those with disabilities. D Nurse advocates make good health care decisions for patients and residents. E Nurse advocates do whatever patients and residents want. F Effective advocacy may entail becoming politically active.

a, c, f. Advocacy is the protection and support of another's rights. Among the patients with special advocacy needs are the very young and the elderly, those who are seriously ill, and those with disabilities; this is not a comprehensive list. Effective advocacy may entail becoming politically active. Patient advocacy is the responsibility of every member of the professional caregiving team—not just nurses. Nurse advocates do not make health care decisions for their patients and residents. Instead, they facilitate their decision making. Advocacy does not entail supporting patients in all their preferences.

Newly hired nurses in a busy suburban hospital are required to read the state Nurse Practice Act as part of their training. Which topics are covered by this act? Select all that apply. A Violations that may result in disciplinary action B Clinical procedures C Medication administration D Scope of practice E Delegation policies F Medicare reimbursement

a, d. Each state has a Nurse Practice Act that protects the public by broadly defining the legal scope of nursing practice. Practicing beyond those limits makes nurses vulnerable to charges of violating the state Nurse Practice Act. Nurse Practice Acts also list the violations that can result in disciplinary actions against nurses. Clinical procedures are covered by the health care institutions themselves. Medication administration and delegation are topics covered by the board of nursing. Laws governing Medicare reimbursement are enacted through Federal legislation.

A registered nurse assumes the role of nurse coach to provide teaching to patients who are recovering from strokes. One example of an intervention the nurse may provide related to this role is: A The nurse uses discovery to identify the patients' personal goals and create an agenda that will result in change. B The nurse is the expert in providing teaching and education strategies to provide dietary and activity modifications. C The nurse becomes a mentor to the patients and encourages them to create their own fitness programs. D The nurse assumes an authoritative role to design the structure of the coaching session and support the achievement of patient goals.

a. A nurse coach establishes a partnership with a patient and, using discovery, facilitates the identification of the patient's personal goals and agenda to lead to change rather than using teaching and education strategies with the nurse as the expert. A nurse coach explores the patient's readiness for coaching, designs the structure of a coaching session, supports the achievement of the patient's desired goals, and with the patient determines how to evaluate the attainment of patient goals.

A nurse is preparing to teach a 45-year-old male patient with asthma how to use his inhaler. Which teaching tool is one of the best methods to teach the patient this skill? A Demonstration B Lecture C Discovery D Panel session

a. Demonstration of techniques, procedures, exercises, and the use of special equipment is an effective patient teaching strategy for a skill. Lecture can be used to deliver information to a large group of patients but is more effective when the session is interactive; it is rarely used for individual instruction, except in combination with other strategies. Discovery is a good method for teaching problem-solving techniques and independent thinking. Panel discussions can be used to impart factual material but are also effective for sharing experiences and emotions.

7. When the attorney representing the patient's family calls Jean and asks to talk with her about the case so that he can better understand her actions, how should Jean respond? A "I'm sorry, but I can't talk with you. You'll have to contact my attorney." B Answer the attorney's questions honestly and make sure that he understands her side of the story. C Appeal to the attorney's sense of compassion and try to enlist his sympathy by telling him how busy it was that morning. D "Why are you doing this to me? This could ruin me!"

a. One of the cardinal rules for nurse defendants is: Do not discuss the case with anyone at your agency (with the exception of the risk manager), with the plaintiff, with the plaintiff's lawyer, with anyone testifying for the plaintiff, or with reporters.

When an older nurse complains to a younger nurse that nurses just aren't ethical anymore, which reply reflects the best understanding of moral development? A "Behaving ethically develops gradually from childhood; maybe my generation doesn't value this enough to develop an ethical code." B "I don't agree that nurses were more ethical in the past. It's a new age and the ethics are new!" C "Ethics is genetically determined ... it's like having blue or brown eyes. Maybe we're evolving out of the ethical sense your generation had." D "I agree! It's impossible to be ethical when working in a practice setting like this!"

a. The ability to be ethical, to make decisions, and to act in an ethically justified manner begins in childhood and develops gradually.

A professional nurse committed to the principle of autonomy would be careful to: A Provide the information and support a patient needed to make decisions to advance one's own interests B Treat each patient fairly, trying to give everyone his or her due C Keep any promises made to a patient or another professional caregiver D Avoid causing harm to a patient

a. The principle of autonomy obligates nurses to provide the information and support patients and their surrogates need to make decisions that advance their interests. Acting with justice means giving each person his or her due, acting with fidelity involves keeping promises to patients, and acting with nonmaleficence means avoiding doing harm to patients.

When instructing a group of older adults preparing for retirement, you would provide information regarding Medicare benefits. Medicare part A is: a) a health insurance program administered locally to cover visits to physicians, medications, and home health. b) an insurance program that pays a set amount based on the diagnosis and approved care for in-hospital services. c) a plan for seniors that limits providers to those within the group plan. d) a federally funded public assistance program.

an insurance program that pays a set amount based on the diagnosis and approved care for in-hospital services. Correct Explanation: The nurse must understand that Medicare is a federal insurance program for the elderly. Medicare part A is based on a prospective payment plan which pays a predetermined fixed amount for in-hospital costs. Medicare part A does not pay physician groups, and is not for the poor, indigent to save on costs.

The nurse manager has noticed a sharp increase in medication errors associated with IV antibiotic administration over the past 2 months. The nurse manager should discuss the situation with each nurse involved and then: a) ask them to attend in-service training for administration of IV medications. b) document it on their evaluations. c) report the incidents to the hospital attorney. d) report them to the supervisor.

ask them to attend in-service training for administration of IV medications. Correct Explanation: Identification of causes of medication errors requires in-service education to inform the staff of strategies to decrease these errors. Errors are frequently the result of systemic problems that can be identified and rectified through problem-solving techniques and changes in procedures. Documenting or reporting the situation would not directly assist the nurses in eliminating errors. Reporting the incidents to the hospital attorney is unnecessary.

A nurse caring for patients in the intensive care unit develops values from experience to form a personal code of ethics. Which statements best describe a characteristic of the development of a personal value system? Select all that apply. A People are born with values. B Values act as standards to guide behavior. C Values are ranked on a continuum of importance. D Values influence beliefs about health and illness. E Value systems are not related to personal codes of conduct. F Nurses should not let their values influence patient care.

b, c, d. A value is a belief about the worth of something, about what matters, which acts as a standard to guide one's behavior. A value system is an organization of values in which each is ranked along a continuum of importance, often leading to a personal code of conduct. A person's values influence beliefs about human needs, health, and illness; the practice of health behaviors; and human responses to illness. Values guide the practice of nursing care. An individual is not born with values; rather, values are formed during a lifetime from information from the environment, family, and culture.

A nurse answers a patient's call light and finds the patient on the floor by the bathroom door. After calling for assistance and examining the patient for injury, the nurse helps the patient back to bed and then fills out an incident report. Which statements accurately describe aspects of this procedure? Select all that apply. A An incident report is used as disciplinary action against staff members. B An incident report is used as a means of identifying risks. C An incident report is used for quality control. D The facility manager completes the incident report. E An incident report makes facts available in case litigation occurs. F Filing of an incident report should be documented in the patient record.

b, c, e. Incident reports are used for quality improvement and should not be used for disciplinary action against staff members. They are a means of identifying risks and are filled out by the nurse responsible for the injured party. An incident report makes facts available in case litigation occurs and in some states, incident reports may be used in court as evidence. A physician completes the incident form with documentation of the medical examination of the patient, employee, or visitor with an actual or potential injury. Documentation in the patient record should not include the fact that an incident report was filed.

A nurse is planning teaching strategies for patients addicted to alcohol, in the affective domain of learning. What are examples of strategies promoting behaviors in this domain? Select all that apply. A The nurse prepares a lecture on the harmful long-term effects of alcohol on the body. B The nurse explores the reasons alcoholics drink and promotes other methods of coping with problems. C The nurse asks patients for a return demonstration for using relaxation exercises to relieve stress. D The nurse helps patients to reaffirm their feelings of self-worth and relate this to their addiction problem. E The nurse uses a pamphlet to discuss the tenants of the Alcoholics Anonymous program to patients. F The nurse reinforces the mental benefits of gaining self-control over an addiction.

b, d, f. Affective learning includes changes in attitudes, values, and feelings (e.g., the patient expresses renewed self-confidence to be able to give up drinking). Cognitive learning involves the storing and recalling of new knowledge in the brain, such as the learning that occurs during a lecture or by using a pamphlet for teaching. Learning a physical skill involving the integration of mental and muscular activity is called psychomotor learning, which may involve a return demonstration of a skill.

A nurse who is working in a hospital setting after graduation from a local college uses value clarification to help understand the values that motivate patient behavior. Which examples denote "prizing" in the process of values clarification? Select all that apply. A A patient decides to quit smoking following a diagnosis of lung cancer. B A patient shows off a new outfit that she is wearing after losing 20 pounds. C A patient chooses to work fewer hours following a stress-related myocardial infarction. D A patient incorporates a new low-cholesterol diet into his daily routine. E A patient joins a gym and schedules classes throughout the year. F A patient proudly displays his certificate for completing a marathon.

b, f. Prizing something one values involves pride, happiness, and public affirmation, such as losing weight or running a marathon. When choosing, one chooses freely from alternatives after careful consideration of the consequences of each alternative, such as quitting smoking and working fewer hours. Finally, the person who values something acts by combining choice into one's behavior with consistency and regularity on the value, such as joining a gym for the year and following a low-cholesterol diet faithfully.

. A nurse who is caring for a morbidly obese male teenager forms a contractual agreement with him to achieve his weight goals. Which statement best describes the nature of this agreement? A "This agreement forms a legal bond between the two of us to achieve your weight goals." B "This agreement will motivate the two of us to do what is necessary to meet your weight goals." C "This agreement will help us determine what learning outcomes are necessary to achieve your weight goals." D "This agreement will limit the scope of the teaching session and make stated weight goals more attainable."

b. A contractual agreement is a pact two people make setting out mutually agreed-on goals. Contracts are usually informal and not legally binding. When teaching a patient, such an agreement can help motivate both the patient and the teacher to do what is necessary to meet the patient's learning outcomes. The agreement notes the responsibilities of both the teacher and the learner, emphasizing the importance of the mutual commitment.

5. If review of this patient's record revealed that she had never consented to the eye surgery, of which intentional tort might the surgeon have been guilty? A Assault B Battery C Invasion of privacy D False imprisonment

b. Assault is a threat or an attempt to make bodily contact with another person without that person's consent. Battery is an assault that is carried out. Every person is granted freedom from bodily contact by another person unless consent is granted. The Fourth Amendment gives citizens the right of privacy and the right to be left alone; a nurse who disregards these rights is guilty of invasion of privacy. Unjustified retention or prevention of the movement of another person without proper consent can constitute false imprisonment.

Janie wants to call an ethics consult to clarify treatment goals for a patient no longer able to speak for himself. She believes his dying is being prolonged painfully. She is troubled when the patient's doctor tells her that she'll be fired if she raises questions about his care or calls the consult. This is a good example of: A Ethical uncertainty B Ethical distress C Ethical dilemma D Ethical residue

b. Ethical distress results from knowing the right thing to do but finding it almost impossible to execute because of institutional or other constraints (in this case, fear of losing her job). Ethical uncertainty results from feeling troubled by a situation but not knowing if it is an ethical problem. Ethical dilemmas occur when the principles of bioethics justify two or more conflicting courses of action. Ethical residue is what nurses experience when they seriously compromise themselves or allow themselves to be compromised.

A nurse is caring for a 42-year-old male patient who is admitted to the hospital with injuries sustained in a motor vehicle accident. While he is in the hospital, his wife tells him that the bottom level of their house flooded, damaging their belongings. When the nurse enters his room, she notes that the patient is visibly upset. The nurse is aware that the patient will most likely be in need of which type of counseling? A Long-term developmental B Short-term situational C Short-term motivational D Long-term motivational

b. Short-term counseling might be used during a situational crisis, which occurs when a patient faces an event or situation that causes a disruption in life, such as a flood. Long-term counseling extends over a prolonged period; a patient experiencing a developmental crisis, for example, might need long-term counseling. Motivational interviewing is an evidence-based counseling approach that involves discussing feelings and incentives with the patient. A caring nurse can motivate patients to become interested in promoting their own health.

A nursing student asks the charge nurse about legal liability when performing clinical practice. Which statement regarding liability is true? A Students are not responsible for their acts of negligence resulting in patient injury. B Student nurses are held to the same standard of care that would be used to evaluate the actions of a registered nurse. C Hospitals are exempt from liability for student negligence if the student nurse is properly supervised by an instructor. D Most nursing programs carry group professional liability making student personal professional liability insurance unnecessary.

b. Student nurses are held to the same standard of care that would be used to evaluate the actions of a registered nurse. Student nurses are responsible for their own acts of negligence if these result in patient injury. A hospital may also be held liable for the negligence of a student nurse enrolled in a hospital-controlled program because the student is considered an employee of the hospital. Nursing instructors may share responsibility for damages in the event of patient injury if an assignment called for clinical skills beyond a student's competency or the instructor failed to provide reasonable and prudent clinical supervision. Most nursing programs require students to carry personal professional liability insurance.

A professional nurse with a commitment to social justice is most apt to: A Provide honest information to patients and the public B Promote universal access to health care C Plan care in partnership with patients D Document care accurately and honestly

b. The American Association of Colleges of Nursing lists promoting universal access to health care as an example of social justice. Providing honest information and documenting care accurately and honestly are examples of integrity, and planning care in partnership with patients is an example of autonomy.

A nurse has taught a diabetic patient how to administer his daily insulin. The nurse should evaluate the teaching-learning process by: A Determining the patient's motivation to learn B Deciding if the learning outcomes have been achieved C Allowing the patient to practice the skill he has just learned D Documenting the teaching session in the patient's medical record

b. The nurse cannot assume that the patient has actually learned the content unless there is some type of proof of learning. The key to evaluation is meeting the learner outcomes stated in the teaching plan.

A nurse is teaching patients of all ages in a hospital setting. Which examples demonstrate teaching that is appropriately based on the patient's developmental level? Select all that apply. A The nurse plans long teaching sessions to discuss diet modifications for an older adult diagnosed with type 2 diabetes. B The nurse recognizes that a female adolescent diagnosed with anorexia is still dependent on her parents and includes them in all teaching sessions. C The nurse designs an exercise program for a sedentary older adult male patient based on the activities he prefers. D The nurse includes an 8-year-old patient in the teaching plan for managing cystic fibrosis. E The nurse demonstrates how to use an inhaler to an 11-year-old male patient and includes his mother in the session to reinforce the teaching. F The nurse continues a teaching session on STIs for a sexually active male adolescent despite his protest that "I've heard enough already!"

c, d, e. Successful teaching plans for older adults incorporate extra time, short teaching sessions, accommodation for sensory deficits, and reduction of environmental distractions. Older adults also benefit from instruction that relates new information to familiar activities or information. School-aged children are capable of logical reasoning and should be included in the teaching-learning process whenever possible; they are also open to new learning experiences but need learning to be reinforced by either a parent or health care provider as they become more involved with their friends and school activities. Teaching strategies designed for an adolescent patient should recognize the adolescent's need for independence, as well as the need to establish a trusting relationship that demonstrates respect for the adolescent's opinions.

4. Jean's attorney was careful to explain in her defense that Jean had specialty knowledge, experience, and clinical judgment and had met certain criteria established by a nongovernmental association, as a result of which she was granted recognition in a specified practice area. What is this sort of credential called? A Accreditation B Licensure C Certification D Board approval

c. Certification is the process by which a person who has met certain criteria established by a nongovernmental association is granted recognition in a specified practice area. Nursing is one of the groups operating under state laws that promote the general welfare by determining minimum standards of education through accreditation of schools of nursing. Licensure is a legal document that permits a person to offer to the public skills and knowledge in a particular jurisdiction, where such practice would otherwise be unlawful without a license. State board of approval ensures that nurses have received the proper training to practice nursing.

3. Those bringing the charges against Jean are called: A Appellates B Defendants C Plaintiffs D Attorneys

c. The person or government bringing suit against another is called the plaintiff. Appellates are courts of law, defendants are the ones being accused of a crime or tort, and attorneys are the lawyers representing both the plaintiff and defendant.

A nurse is teaching a 50-year-old male patient how to care for his new ostomy appliance. Which teaching aid would be most appropriate to confirm that the patient has learned the information? A Ask Me 3 B Newest Vital Sign C Teach-back tool D TEACH acronym

c. The teach-back tool is a method of assessing literacy and confirming that the learner understands health information received from a health professional. The Ask Me 3 is a brief tool intended to promote understanding and improve communication between patients and their providers. The Newest Vital Sign (NVS) is a reliable screening tool to assess low health literacy, developed to improve communications between patients and providers. The TEACH acronym is used to maximize the effectiveness of patient teaching by tuning into the patient, editing patient information, acting on every teaching moment, clarifying often, and honoring the patient as a partner in the process.

A state attorney decides to charge a nurse with manslaughter for allegedly administering a lethal medication. This is an example of what type of law? A Public law B Private law C Civil law D Criminal law

d. Criminal law concerns state and federal criminal statutes, which define criminal actions such as murder, manslaughter, criminal negligence, theft, and illegal possession of drugs. Public law regulates relationships between people and the government. Private or civil law includes laws relating to contracts, ownership of property, and the practice of nursing, medicine, pharmacy, and dentistry.

6. What must be established to prove that malpractice or negligence has occurred in this case? A The surgeon who performed the procedure called the nurse's action pure negligence, saying that the patient could have been saved. B The fact that this patient should not have died—she was a vibrant grandmother of 10 who had walked three quarters of a mile the morning of her surgery and had sung in her church choir the day before. C The nurse intended to harm the patient and was willfully negligent. D The nurse had a duty to monitor the patient's vital signs, failed to do so, the patient died, and it was Jean's failure to do her duty that caused the patient's death.

d. Liability involves four elements that must be established to prove that malpractice or negligence has occurred: duty, breach of duty, causation, and damages. Duty refers to an obligation to use due care (what a reasonably prudent nurse would do) and is defined by the standard of care appropriate for the nurse-patient relationship. Breach of duty is the failure to meet the standard of care. Causation, the most difficult element of liability to prove, shows that the failure to meet the standard of care (breach) actually caused the injury. Damages are the actual harm or injury resulting to the patient.

A nurse administers the wrong medication to a patient and the patient is harmed. The physician who ordered the medication did not read the documentation that the patient was allergic to the drug. Which statement is true regarding A liability for the administration of the wrong medication? B The nurse is not responsible, because the nurse was merely following the doctor's orders. C Only the nurse is responsible, because the nurse actually administered the medication. D Only the physician is responsible, because the physician actually ordered the drug. E Both the nurse and the physician are responsible for their respective actions.

d. Nurses are legally responsible for carrying out the orders of the physician in charge of a patient unless an order would lead a reasonable person to anticipate injury if it were carried out. If the nurse should have anticipated injury and did not, both the prescribing physician and the administering nurse are responsible for the harms to which they contributed.

The American Association of Colleges of Nursing identified five values that epitomize the caring professional nurse. Which of these is best described as acting in accordance with an appropriate code of ethics and accepted standards of practice? A Altruism B Autonomy C Human dignity D Integrity

d. The American Association of Colleges of Nursing defines integrity as acting in accordance with an appropriate code of ethics and accepted standards of practice. Altruism is a concern for the welfare and well-being of others. Autonomy is the right to self-determination, and human dignity is respect for the inherent worth and uniqueness of individuals and populations.

A nurse is counseling a 19-year-old male athlete who had his right leg amputated below the knee following a motorcycle accident. During the rehabilitation process, the patient refuses to eat or get up to ambulate on his own. He says to the nurse, "What's the point. My life is over now and I'll never be the football player I dreamed of becoming." What is the nurse counselor's best response to this patient? A "You're young and have your whole life ahead of you. You should focus on your rehabilitation and make something of your life." B "I understand how you must feel. I wanted to be a famous singer, but I wasn't born with the talent to be successful at it." C "You should concentrate on other sports that you could play even with a prosthesis." D "I understand this is difficult for you. Would you like to talk about it now or would you prefer me to make a referral to someone else?"

d. This answer communicates respect and sensitivity to the patient's needs and offers an opportunity to discuss his feelings with the nurse or another health care professional. The other answers do not allow the patient to express his feelings and receive the counseling he needs.

Five-year-old Bobby has dietary modifications related to his diabetes. His parents want him to value good nutritional habits and they decide to deprive him of a favorite TV program when he becomes angry after they deny him foods not on his diet. This is an example of what mode of value transmission? A Modeling B Moralizing C Laissez-faire D Rewarding and punishing

d.When rewarding and punishing are used to transmit values, children are rewarded for demonstrating values held by parents and punished for demonstrating unacceptable values. Through modeling, children learn what is of high or low value by observing parents, peers, and significant others. Children whose caregivers use the moralizing mode of value transmission are taught a complete value system by parents or an institution (e.g., church or school) that allows little opportunity for them to weigh different values. Those who use the laissez-faire approach to value transmission leave children to explore values on their own (no single set of values is presented as best for all) and to develop a personal value system.

Chest tubes

drain air and fluids and re-establish negative pressure tube goes into pleural space Awake and usually in respiratory distress during insertion keep drainage system on floor with flanges taped to floor

A home health nurse who performs a careful safety assessment of the home of a frail elderly patient to prevent harm to the patient is acting in accordance with which of the principles of bioethics? A Autonomy B Beneficence C Justice D Fidelity E Nonmaleficence

e. Nonmaleficence is defined as the obligation to prevent harm. Autonomy is respect for another's right to make decisions, beneficence obligates us to benefit the patient, justice obligates us to act fairly, and fidelity obligates us to keep our promises.

Because of an outbreak of influenza among the nursing staff, the hospital is very short staffed. The nurse manager prioritizes client needs on the surgical unit by which strategy? a) asking unlicensed assistive personnel (UAPs) to assist in administering analgesics b) ensuring that clients receive medications but omitting full bathing when possible c) allowing all medications to be given 2 hours late d) rescheduling surgeries

ensuring that clients receive medications but omitting full bathing when possible Correct Explanation: Daily bathing is not required to meet standards of care. Rescheduling surgeries is not a strategy for meeting nursing care needs of clients. Medications are required to be given as

A student nurse is performing a sterile dressing change on a patient's abdominal incision. While establishing her sterile field, the nurse drops her forceps on the floor and is unable to continue with the dressing change because she has no extra supplies in the room and no one in the room to assist her by bringing new forceps. The student has failed to organize: a) logistics and planning. b) environment and patient. c) equipment and personnel. d) skills and assistance.

equipment and personnel. Correct Explanation: A key component of the organizing interventions is to ensure adequate equipment (extra supplies) and sufficient personnel to assist with more complex tasks.

The nurse has asked the unlicensed assistive personnel (UAP) to ambulate a client with Parkinson's disease. The nurse observes the UAP pulling on the client's arms to get the client to walk forward. The nurse should: a) explain how to overcome a freezing gait by telling the client to march in place. b) give the client a muscle relaxant. c) have the UAP keep a steady pull on the client to promote forward ambulation. d) assist the UAP with getting the client back in bed.

explain how to overcome a freezing gait by telling the client to march in place. Correct Explanation: Clients with Parkinson's disease may experience a freezing gait when they are unable to move forward. Instructing the client to march in place, step over lines in the flooring, or visualize stepping over a log allows them to move forward. It is important to ambulate the client and not keep them on bed rest. A muscle relaxant is not indicated.

When the patient's primary care physician does not have hospital privileges, there is a service to provide the patient's care and communicate to the patient's physician. This service is provided by physicians called: a) hospitalists. b) referral specialists. c) on-call physicians. d) admitting doctors.

hospitalists. Correct Explanation: The hospitalist is a physician who provides care to the patient in the emergency room and after admission to the hospital. The hospitalist communicates with the patient's primary doctor but manages the hospital care

The nurse is developing a primary disease prevention program for older adults. Which topic is the most appropriate? a) diet and exercise for people with heart disease b) immunizations for influenza c) range of motion exercises d) blood glucose screening for diabetes

immunizations for influenza Correct Explanation: The three levels of prevention are primary, secondary, and tertiary. The most important topic to include in a program for older adults that is to emphasize primary prevention is the importance of receiving immunizations to prevent disease such as influenza, pneumonia, and shingles. The goal of primary prevention is to protect healthy people from developing a disease or injury. Immunization is an example of a primary prevention strategy. Secondary prevention involves taking action to slow or stop the progress of a disease. Monitoring blood glucose for clients with diabetes or initiating diet and exercise programs for people with heart disease are examples of secondary prevention. Tertiary preventions are treatments aimed to reduce the negative impact of established disease by restoring function and reducing disease-related complications. An example of tertiary prevention is performing passive and active range of motion exercises to prevent disability.

A client has a reddened area over a bony prominence. The nurse finds an unlicensed nursing personnel (UAP) massaging this area. The nurse should: a) inform the UAP that massage is even more effective when combined with the use of lotion. b) instruct the UAP that massage is contraindicated because it decreases blood flow to the area. c) explain to the UAP that massage is effective because it improves blood flow to the area. d) reinforce the UAP's use of this intervention over the bony prominence.

instruct the UAP that massage is contraindicated because it decreases blood flow to the area. Correct Explanation: Massaging an area that is reddened due to pressure is contraindicated because it further reduces blood flow to the area. In the past, massaging reddened areas was thought to improve blood flow to the area, and some nursing personnel may still believe that massaging the area is effective in preventing pressure ulcer formation

What was one barrier to the development of the nursing profession in the United States after the Civil War? a) lack of influence from nursing leaders b) hospital-based schools of nursing c) independence of nursing orders d) lack of educational standards

lack of educational standards Correct Explanation: A lack of educational standards was one barrier to the development of the nursing profession after the Civil War. Other barriers included a male dominance of healthcare and the pervading belief that women were dependent on men. The location of nursing schools, a lack of influence from nursing leaders, and independent nursing orders were not barriers to the development of the nursing profession after the Civil War

A malpractice case involving an incident in the operating room has entered litigation. A scrub nurse has been asked to present her firsthand knowledge of the incident based upon recollection of the incident and documentation in the patient record. The nurse's role in this malpractice case is: a) nurse as an expert witness. b) nurse as the defendant. c) nurse as a fact witness. d) nurse as the plaintiff.

nurse as a fact witness. Correct Explanation: The nurse will act as a fact witness. Fact witnesses, who are placed under oath, must base their testimony on only firsthand knowledge of the incident and not on assumptions. The nurse will be asked if the testimony is based on independent recollection of the incident or on documentation in the patient's record. The nurse as defendant will work closely with an attorney while preparing the defense. The nurse as an expert witness will explain to the judge and jury what happened based upon the patient's record and offer an opinion about whether the nursing care met acceptable standards

Several hours into a shift, a nurse on a very busy medical-surgical unit privately asks the charge nurse to change her assignment. She is frustrated because she has had to devote so much time and energy to helping a newly licensed nurse provide discharge teaching for clients with diabetes mellitus. The charge nurse should: a) insist that the nurse follow through with the assignment. b) try to provide the staff member with a float nurse. c) offer to assist with the discharge teaching needs. d) reassign the new graduate to another staff member.

offer to assist with the discharge teaching needs. Explanation: Staff members need to know the charge nurse is a supportive leader who respects their honesty and stands behind them. By offering to help with discharge teaching, the charge nurse is actively engaging with her staff at a time of need. Changing all the assignments on this extremely busy floor would be counterproductive. Insisting that the staff member follow through with her assignment disrespects her request and genuine need. Providing a float nurse could help, but there are no guarantees a float nurse is available

The focus of a hospital's current quality assurance program is a comparison between the health status of patients on admission and at the time of discharge. This form of quality assurance is characteristic of: a) process evaluation. b) nursing audit. c) structure evaluation. d) outcome evaluation.

outcome evaluation. Correct Explanation: Outcome evaluation focuses on measurable changes in the health status of the patient or the end results of nursing care. Whereas the proper environment for care and the right nursing actions are important aspects of quality care, the critical element in evaluating care is demonstrable changes in patient health status. Process evaluation addresses performance expectations during the various stages of the nursing process. Structure evaluation addresses the environment of care and a nursing audit focuses on the review of records

An alert and oriented older adult female with metastatic lung cancer is admitted to the medical-surgical unit for treatment of heart failure. She was given 80 mg of furosemide in the emergency department. Although the client is ambulatory, the unlicensed assistive personnel (UAP) are concerned about urinary incontinence because the client is frail and in a strange environment. The nurse should instruct the UAP to assist with implementing the nursing plan of care by: a) requesting an indwelling urinary catheter to avoid incontinence b) placing a commode at the bedside and instructing the client in its use c) padding the bed with extra absorbent linens d) prescribing adult diapers for the client so she will not have to worry about incontinence

placing a commode at the bedside and instructing the client in its use Correct Explanation: A bedside commode should be near the client for easy, safe access. Measurement of urine output is also important in a client with heart failure. Putting diapers on an alert and oriented individual would be demeaning and inappropriate. Indwelling catheters are associated with increased risk of infection and are not a solution to possible incontinence. There is no reason to think that the client would not be able to use the bedside commode.

A client is being prepared for a bronchoscopy. The nurse can delegate which task to the unlicensed assistive personnel (UAP)? a) instructing the client about the procedure b) evaluating the client's level of anxiety c) placing the client on NPO status d) obtaining the signed consent form

placing the client on NPO status Correct Explanation: It would be appropriate for the nurse to instruct the UAP to place the client on NPO status. It is the responsibility of the health care provider performing the procedure to obtain

A type of nursing which includes individualized care provided by the same nurse throughout the period of care is defined as a) functional nursing. b) patient-focused nursing. c) primary nursing. d) case management.

primary nursing. Correct Explanation: Primary nursing refers to comprehensive, individualized care provided by the same nurse throughout the period of care. Case management is not a type of nursing care delivery. Patient-focused nursing is characterized by assigning a nurse to manage the care of a caseload of patients during a given shift, who may then delegate activities to other nursing personnel. Functional nursing is not individualized to the extent that one nurse cares for the patient throughout the entire period of care

An unlicensed assistive personnel (UAP) is providing care to a client with left-sided paralysis. Which action by the UAP requires the nurse to provide further instruction? a) providing passive range of motion exercises to the left extremities during the bed bath b) pulling up the client under the left shoulder when getting the client out of bed to a chair c) elevating the foot of the bed to reduce edema d) putting high top tennis shoes on the client after bathing

pulling up the client under the left shoulder when getting the client out of bed to a chair Correct Explanation: Pulling the client up under the arm can cause shoulder displacement. A belt around the waist should be used to move the client. Passive range-of-motion exercises prevent contractures and atrophy. Raising the foot of the bed assists in venous return to reduce edema. High top tennis shoes are used to prevent foot drop

A prescription has just been received for a 72-year-old client with gastrointestinal hemorrhage to have two blood transfusions. The registered nurse caring for the client is a pediatric nurse temporarily assigned to the unit who has never administered blood before. The best action of the charge nurse is to: a) ask the nurse to determine how confident he or she is to administer the blood safely. b) give a thorough explanation of the procedure for blood administration to the nurse. c) ask the nurse to read the policy book before administering the blood. d) reassign the client to another nurse who is experienced in blood administration.

reassign the client to another nurse who is experienced in blood administration. Correct Explanation: The best option in this situation is to reassign the client to a nurse with experience in blood administration. The policy book and explanation are resources, but the nurse is a pediatric nurse who has never administered blood before, and therefore, an unsafe situation is created. An explanation is insufficient teaching for safe and proper blood administration, and reading policy book may be a resource, but having an experienced nurse administer the blood is a safer decision. Asking about the nurse's confidence is not sufficient evidence that the nurse can administer the blood. Asking an experienced nurse to administer the blood is a safer option.

A nurse working in a physician's office uses the managerial function known as "organizing." What is involved in this function? a) problems b) evaluation c) workforce d) resources

resources Correct Explanation: The organizing function of the nurse manager involves acquiring, managing, and mobilizing resources to meet both clinical and financial objectives.

When the nurse observes that the patient's heart rate increases during inspiration and decreases during expiration, the nurse reports that the patient is demonstrating a) normal sinus rhythm. b) sinus bradycardia. c) sinus dysrhythmia. d) sinus tachycardia.

sinus dysrhythmia. Explanation: Sinus dysrhythmia occurs when the sinus node creates an impulse at an irregular rhythm. Normal sinus rhythm occurs when the electrical impulse starts at a regular rate and rhythm in the SA node and travels through the normal conduction pathway. Sinus bradycardia occurs when the sinus node regularly creates an impulse at a slower-than-normal rate. Sinus tachycardia occurs when the sinus node regularly creates an impulse at a faster-than-normal rate

A client is being discharged from the acute inpatient unit but needs further continuous supervision that is less intense than inpatient hospitalization. The nurse should refer the client to which setting? a) subacute unit b) partial program c) group home. d) daily outpatient visits

subacute unit Explanation: When inpatient hospitalization is no longer needed, subacute care is the next least restrictive setting when the client requires 24-hour supervision but less extensive and intensive services. Subacute units provide the client with a bed, meals, medication, groups, and activities. The client has autonomy and independence in choosing which groups to attend and can seek employment and housing and apply to school or training. A partial program provides structured activities and ongoing treatment from 4 to 8 hours per day and 1 to 5 days per week. Daily outpatient visits would provide counseling for a small period of time. A group home, temporary or permanent, would provide some supervision, a bed, meals, and laundry facilities. Some group homes provide group therapy and structured activities. However, this would not provide an appropriate level of care for a client who requires 24-hour supervision, structure, and programming.

The termination stage of the Transtheoretical Model of Change occurs when a) the person constructs a plan to change behavior. b) the person is not thinking about making a change. c) the person takes steps to operationalize the plan of action. d) the person has the ability to resist relapsing back to unhealthy behavior.

the person has the ability to resist relapsing back to unhealthy behavior. Correct Explanation: The termination stage of the Transtheoretical Model of Change occurs when a person has the ability to resist relapsing back to unhealthy behavior. Operationalizing a plan of action, constructing a plan to change behavior, and not thinking about making a change are not part of the termination stage

The termination stage of the Transtheoretical Model of Change occurs when a) the person has the ability to resist relapsing back to unhealthy behavior. b) the person is not thinking about making a change. c) the person constructs a plan to change behavior. d) the person takes steps to operationalize the plan of action.

the person has the ability to resist relapsing back to unhealthy behavior. Explanation: The termination stage of the Transtheoretical Model of Change occurs when a person has the ability to resist relapsing back to unhealthy behavior. Operationalizing a plan of action, constructing a plan to change behavior, and not thinking about making a change are not part of the termination stage

An airplane crash results in mass casualties. The nurse is directing personnel to tag all victims. Which information should be placed on the tag? Select all that apply. a) presence of jewelry b) medications and treatments administered c) identifying information when possible (such as name, age, and address) d) triage priority e) next of kin

triage priority • identifying information when possible (such as name, age, and address) • medications and treatments administered Explanation: Tracking victims of disasters is important for casualty planning and management. All victims should receive a tag, securely attached, that indicates the triage priority, any available identifying information, and what care, if any, has been given along with time and date. Tag information should be recorded in a disaster log and used to track victims and inform families. It is not necessary to document the presence of jewelry or next of kin.

During electrical cardioversion, the defibrillator is set to synchronize with the ECG so that the electrical impulse discharges during a) ventricular repolarization. b) ventricular depolarization. c) atrial depolarization. d) the QT interval.

ventricular depolarization. Correct Explanation: In cardioversion, the defibrillator is set to synchronize with the ECG on the cardiac monitor so that the electrical impulse discharges during ventricular depolarization.

A home care nurse is preparing a presentation for a group of nursing students about the area of practice. Which statement would the nurse most likely include when describing this practice area? a) "Home care requires that the nurse has an advanced degree in nursing." b) "Home care nurses need to have high-level assessment skills." c) "The major emphasis of care involves accessing community resources." d) "Decision-making is shared among the other health care professionals."

"Home care nurses need to have high-level assessment skills." Correct Explanation: A nurse working in home care needs advanced knowledge and skills in general nursing practice with an emphasis on community health and acute medical-surgical nursing. The home care nurse also needs high-level assessment, critical thinking, and decision-making skills in a setting where other health care professionals are not available to validate observations, conclusions, and decisions. Home care nurses provide "high-tech, high-touch" care to those with acute health care needs. They are responsible for client and family teaching, for contacting community resources, and coordinating the continuing care of clients. An advanced nursing degree is not necessarily required

A nurse is caring for a client who has had a massive stroke. The family communicates concern about the actions of the nurse on the previous shift. The family reports that the nurse didn't administer medications properly or maintain client privacy. When responding to complaints about a colleague, the nurse should say: a) "I'd be happy to get the charge nurse to see what we can do for the client." b) "We've had some problems with her lately. She never seems to get to work on time." c) "I understand your concern, but I'm sure she didn't do those things." d) "I think you're right. I'll let the physician know this has happened so that nurse will get in trouble."

"I'd be happy to get the charge nurse to see what we can do for the client." Correct Explanation: The nurse should follow the facility's policy and chain of command for handling complaints, which commonly begins with the charge nurse. The nurse should avoid trying to handle a problem involving another employee because doing so will only aggravate the issue. The nurse shouldn't side with the family or with the other nurse.


Ensembles d'études connexes

6531 Combined midterm and final review

View Set

AMSCO Chapter 17 - Measures of Development

View Set

Unit 20 - Controls & Instrumentation (Ch. 84-89)

View Set

Economics: Section Reviews for Chapter Thirteen

View Set

BY662 - Lecture 23 - Sensitization and Habituation (Learning)

View Set